[obm-l] Dilema dos prisioneiros

2010-04-25 Por tôpico Douglas Ribeiro Silva
Olá pessoal!

Estou fazendo o curso de Sistemas Probabilísticos e gostaria de uma
ajuda com o seguinte problema:

De 3 prisioneiros, A, B e C, serão escolhidos 2 para ser libertados. O
prisioneiro A pensa em perguntar ao carcereiro se ele será libertado,
mas acha que não é ético da parte dele. Então ele pensa em perguntar o
nome do outro prisioneiro que será libertado. No entanto, ele desiste
de perguntar, pois após fazer a pergunta, A acha que a probabilidade
dele ser solto passa de 2/3 para 1/2. Mostre porque o raciocício de A
está errado.

Até mais!

=
Instruções para entrar na lista, sair da lista e usar a lista em
http://www.mat.puc-rio.br/~obmlistas/obm-l.html
=


Re: [obm-l] Jogo dos 4 bits

2008-11-22 Por tôpico Douglas Ribeiro Silva
Olá pessoal! Desculpem a minha ausencia esses dias da lista pra
responder às duvidas dos que responderam o meu e-mail inicial.

Ótima estratégia Ralph! Gostei bastente mesmo do seu método! Mas como
você mesmo levantou a hipótese, será que da pra fazer com menos
tentativas?

Acho que não... pelos motivos citados pelo Maurício

Acho que chegamos então que o mínimo são 5 tentativas.. Obrigado a todos!


2008/11/18 Ralph Teixeira [EMAIL PROTECTED]:
 Se o objetivo eh minimizar o numero **maximo** de palpites... Certamente, eh
 possivel adivinhar em um maximo de 5 palpites, usando a seguinte estrategia
 de ir trocando um digito de cada vez (Pi=i-esimo palpite, Ri=i-esima
 resposta):

 P1=
 P2=0001
 P3=0011
 P4=0111

 Se a resposta melhorou ao passar de Pi para Pi+1, eh porque aquele digito
 que voce trocou estah correto, e vice-versa. Ou seja, apos estes 4 palpites,
 voce jah sabe os ultimos 3 digitos com certeza.
 Agora basta olhar a resposta a P1 para descobrir se o digito incerto eh 0 ou
 1; assim, o 5o palpite serah correto.

 Exemplo:
 R1=1, R2=2, R3=1 e R4=2.
 Como R2R1, o ultimo digito eh 1, isto eh, xxx1 (pois esta eh a unica
 diferenca entre P1 e P2);
 Como R3R2, xx01.
 Como R4R3, x101.
 Enfim, como R1=1, soh tem um 0 na resposta, entao 1101 eh a resposta.

 Esta estrategia eh facilmente generalizavel: sempre eh possivel adivinhar um
 numero de n bits com, no maximo, n+1 palpites (agora, serah que dah com
 menos?).

 Abraco,
   Ralph

 2008/11/18 Douglas Ribeiro Silva [EMAIL PROTECTED]

 O jogo dos 4 bits consiste no computador escolher um número de 4 bits
 e o usuário tentar adivinhar. Para cada palpite do usuário o
 computador retorna quantos bits ele acertou.

 Ex: o computador escolhe 0101

 Usuario: 
 PC:2
 Usuario: 0100
 PC: 3
 Usuario: 
 PC: 2
 Usuario: 0111
 PC: 1
 Usuario: 0101
 PC: 4

 Qual a melhor estratégia para o jogo? O jogador deve sempre trocar a
 quantidade de dígitos que o computador indicar? Qual a quantidade
 máxima que um usuário inteligente gastaria para acertar o numero?

 =
 Instruções para entrar na lista, sair da lista e usar a lista em
 http://www.mat.puc-rio.br/~obmlistas/obm-l.html
 =



=
Instruções para entrar na lista, sair da lista e usar a lista em
http://www.mat.puc-rio.br/~obmlistas/obm-l.html
=


[obm-l] Jogo dos 4 bits

2008-11-17 Por tôpico Douglas Ribeiro Silva
O jogo dos 4 bits consiste no computador escolher um número de 4 bits
e o usuário tentar adivinhar. Para cada palpite do usuário o
computador retorna quantos bits ele acertou.

Ex: o computador escolhe 0101

Usuario: 
PC:2
Usuario: 0100
PC: 3
Usuario: 
PC: 2
Usuario: 0111
PC: 1
Usuario: 0101
PC: 4

Qual a melhor estratégia para o jogo? O jogador deve sempre trocar a
quantidade de dígitos que o computador indicar? Qual a quantidade
máxima que um usuário inteligente gastaria para acertar o numero?

=
Instruções para entrar na lista, sair da lista e usar a lista em
http://www.mat.puc-rio.br/~obmlistas/obm-l.html
=


[obm-l] Convolução

2008-04-15 Por tôpico Douglas Ribeiro Silva
Olá pessoal!

Alguém tem indicação de algum site ou bom livro que dê maiores
detalhes sobre aplicações da integral de convolução? Agradeço qualquer
ajuda

Abraços!

=
Instruções para entrar na lista, sair da lista e usar a lista em
http://www.mat.puc-rio.br/~obmlistas/obm-l.html
=


Re: [obm-l] Modelos matemáticos para auxiliar no combate à dengue

2008-04-02 Por tôpico Douglas Ribeiro Silva
Olá Marcelo!

Sugiro que entre em contato com o professor César Castilho do
Departamento de Matemática da UFPE, o qual já orientou trabalhos
relativos a dengue. Segue abaixo o curriculo lattes dele e a pagina no
departamento, que por sinal contem um link falando sobre a modelagem
do problema

http://buscatextual.cnpq.br/buscatextual/visualizacv.jsp?id=K4721454A6
http://www.dmat.ufpe.br/~castilho/index.html

Abraços

Em 02/04/08, Marcelo Salhab Brogliato[EMAIL PROTECTED] escreveu:
 Olá para todos,

 peço ajuda aos amigos, e qualquer idéia é muito bem vinda.
 Criar um modelo, que, dado a posição da casa e do trabalho das pessoas
 contaminadas com dengue, tenta prever os possíveis focos do mosquito. Deste
 modo, auxiliando o combate contra esta praga que hoje está presente aqui no
 Rio de Janeiro.

 Pensei em uma modelo probabilístico, no qual serão geradas funções de
 probabilidade do foco do mosquito para cada pessoa, e será analisada a
 combinação linear destas funções, com pesos diferenciados para as pessoas.
 Por exemplo, uma pessoa que só fica em casa, deve ter um peso maior para a
 posição de sua casa, já uma pessoa que fica o dia todo em trânsito, deve ter
 um peso bem reduzido em tudo, já que pode ter contaminado a doença em
 qualquer local.

 Estou tentando descobrir sobre o raio de ação do mosquito, horários que ele
 está mais ativo (encontrei que é pelo período da manhã, mas informação de
 internet nem sempre é correta.. estou tentando descobrir de alguns centros
 de estudo), e quaisquer outras informações que possam ajudar no modelo.

 Com um pouco mais de matemática, vamos tentar simplificar o problema
 original. Suponhamos que uma mosquito consiga sobrevoar uma circunferência
 de raio R, e que exista apenas um mosquito. Temos um conjunto de duplas
 (c[k], t[k]), pertencentes ao C^2, que correspondem as posições no plano de
 onde moram e trabalham pessoas picadas por este mosquito. Suponhamos também,
 que cada pessoa fica a[k] por cento do tempo em casa, b[k] por cento do
 tempo no trabalho e c[k] por cento do tempo em outros locais. Crie um modelo
 que encontre, com certa probabilidade, o centro da circunferência de alcance
 do mosquito.

 agradeço desde já,
 abraços,
 Salhab



=
Instruções para entrar na lista, sair da lista e usar a lista em
http://www.mat.puc-rio.br/~obmlistas/obm-l.html
=


[obm-l] OFF TOPIC - Receberam o e-mail anterior?

2007-11-13 Por tôpico Douglas Ribeiro Silva
Olá pessoal!

Com toda a discussão sobre problemas no e-mail, eu gostaria de saber
se vocês receberam um e-mail anterior meu com o título Quadrando
quadrados + teorema das 4 cores.
Conta aqui no gmail na caixa de itens enviados, mas se foi respondido
eu não recebi nada.
Aliás, alguém ao menos pensou no caso? Em caso positivo, se não quiser
se manifestar na lista pode me mandar um e-mail privado.

Desculpem qualquer incomodo.

Abraços,

Douglas Ribeiro

=
Instruções para entrar na lista, sair da lista e usar a lista em
http://www.mat.puc-rio.br/~obmlistas/obm-l.html
=


[obm-l] Quadrando Quadrados + Teorema das 4 Cores

2007-11-11 Por tôpico Douglas Ribeiro Silva
Olá pessoal!

Estive olhando esses dias na internet algumas páginas que falam sobre
o problema de quadrar quadrados. Aos que não estão familiarizados com
o assunto sugiro
http://mathworld.wolfram.com/PerfectSquareDissection.html

Com isso surgiu a vontade de criar uma réplica do 21 square perfect
square usando cartolina ou algo do tipo e além disso usar o teorema
das 4 cores. Sobre o teorema das 4 cores:
http://en.wikipedia.org/wiki/Four_color_theorem

Agora é que vem a minha pergunta:

Como fazer a distribuição das cores de modo que a diferença de áreas
entre as cores seja mínima?

=
Instruções para entrar na lista, sair da lista e usar a lista em
http://www.mat.puc-rio.br/~obmlistas/obm-l.html
=


Re: [obm-l] Off topic... quase...

2007-11-11 Por tôpico Douglas Ribeiro Silva
Olá pessoal!
Aos que usam gmail, tentem olhar a pasta E-mails enviados. Tambémnao recebo 
na pasta caixa de entrada as minhas proprias mensagens, masmuitas vezes quando 
eu envio uma mensagem à lista e ela é respondida,não faço idéia do motivo, mas 
as respostas a ela estão juntamente coma mensagem original dentro da pasta de 
E-mails enviados. Como disse oNehab... prefiro acreditar que é alguma bruxaria 
determinística.
Abraços!
Em 09/11/07, Palmerim Soares[EMAIL PROTECTED] escreveu: O problema é quando 
a mensagem que você envia, por algum motivo, não chega à lista (o que acontece 
algumas vezes). Como não vem nenhum aviso do gmail dizendo que sua mensagem 
não chegou ao destino, fica difícil saber se a mensagem chegou e ninguém deu a 
mínima para ela, ou se não chegou e por isso não houve resposta de ninguém. 
Isso já aconteceu comigo algumas vezes aqui na lista. Palmerim Em 
09/11/07, Daniel S. Braz [EMAIL PROTECTED] escreveu:  Jones e demais,  
Bem, não foi exatamente uma explicação...mas...O que acontece é que ele mantém 
apenas uma cópia de cada mensagem.Experimente mandar um e-mail para você 
mesmo, quando recebe-lo em suacaixa de entrada, delete-o, verifique que ele 
também foi apagado desua caixa de saída. Ou seja, na verdade ele não recebeu 
a mensagemque você enviou para você mesmo e sim disponibilizou-a também na 
suacaixa de entrada (ele!
 mantém apenas uma cópia de cadamensagem!..analisando o header da mesma). O 
mesmo acontece quandoenviamos a mensagem para a lista, como você já possui a 
mensagem (emsua caixa de saída) o gmail não receberá a mesma 
mensagem...issotudo para manter coerente o layout chat dos e-mail (mantendo 
todo ohistórico das conversar na mesma página).  Nehab, como você deve ter 
percebido, você conseguirá ver sua mensagemna sua caixa de entrada assim que 
alguém responde-la (logo acima daresposta).  abraços,  On Nov 9, 2007 6:03 
PM, jones colombo  [EMAIL PROTECTED] wrote: Daniel, esta explicação não 
convence, visto que se eu  envio um email para mim mesmo,  eu recebo o email, 
ou seja, ele pode até analizar o header mas não usa esta análise desta 
maneira.   Jones  On Nov 9, 2007 12:38 PM, Daniel S. Braz  [EMAIL 
PROTECTED] wrote:  Oi Nehab,  O Palmerim está certo. Eu também uso gmail 
e também não recebo asmensagens que envio*!
* para a lista. Ou seja, é realmente umacaracteristica dele!
 e não falta (ou erro) de configuração.  ** Na verdade, creio eu,  a 
mensagem é recebida, mas não é exibidapelo gmail. Ele deve analisar o header 
e se a mensagem for a mesma quevocê enviou, ele descarta (já que você possui 
a mesma mensagem em seue-mail, só que na caixa de saída).  abraços,  
Daniel.  --  On 11/9/07, Carlos Nehab  [EMAIL PROTECTED] wrote: Oi, 
Palmerim e Teresa e outros (eventualmente torturados que nem eu). Começo a 
não !  achar este tópico tão off-topic assim... Não acredito no 
não-determinismo do gmail (nem em sua aleatoriedade - no péssimo sentido).   
Espero não ter que concordar com você :-)   Breve voltarei ao assunto... 
Abração e obrigado pelas palavras de consolo :-) Nehab PS1: Eu ainda 
acho que tem o que haver com email de envio diferente de email de retorno 
e de configuração de estratégias de proteção contra spams. Veja em 
http://www.greatcircle.com/majordomo!
/majordomo-faq.html PS2:  Antes eu recebia emails dobrados e agora que 
descadastrei  os outros emails e fiquei só com o gmail, não recebo meu 
próprio email...  Mas ainda tenho esperaça de que é uma bruxaria 
deterministica... Aguardemos... Palmerim Soares escreveu: Mestre 
Nehab, Saudações! O gmail tem essa característica mesmo, o que é p!   
éssimo, pois você não sabe se o pessoal da lista recebeu ou não a sua 
mensagem. Pelo que eu sei através de conversas !  com outros colegas que 
usam gmail, não é o caso de config!  uração, e, portanto, não tem jeito. 
Abraços Palmerim Em 09/11/07, Carlos Nehab  [EMAIL PROTECTED] 
escreveu:  Desculpem a insistência com os testes,   Colegas tem 
recebido minhas mensagens, mas eu . não !!!  Por isto  mais um teste 
para eu tentar descobrir o mistério...   Nehab  
=  
Instruç�!
�es para entrar na lista, sair da lista e usar a lista em !
 http://www.mat.puc-rio.br/~obmlistas/obm-l.html  
 = 
  
 = 
 Instru�ões para entrar na lista, sair da lista e usar a lista em 
 http://www.mat.puc-rio.br/~obmlistas/obm-l.html !  
 =   -- O modo 
 mais provável do mundo ser destruído, como concordam a maioriados 
 especialistas, é atra!  vés de um acidente. É aí que nós entramos.Somos 
 profissionais da computação. Nós causamos acidentes - NathanielBorenstein 
  
 = 
  Instru�ões para entrar na lista, 

Re: [obm-l] Cubo de Rubik

2007-09-12 Por tôpico Douglas Ribeiro Silva
Olá Pedro!

Eu tenho uma idéia de onde vieram alguns números mas não entendo
perfeitamente a formula como um todo... vamos lá:

8!12!(3^8)(2^12)/2*3*2

Os numeros do numerador surgem do fato que o cubo possui 27 cubos
menores. O cubo central não conta para as possiveis permutacoes, logo
temos 26. Note também que o cubo que representa o centro de cada face
é fixo, por isso temos 6 cubos que não irão participar da contagem.
Logo resta 20 cubos.
Desses 20, 8 são cubos de quina, que possuem 3 cores e 12 são cubos de
aresta que possuem duas cores tenho certeza que é daí que surgem o 8!
12! 3^8 e 2^12

O restante eu não entendi muito bem o que foi feito e gostaria que
alguém explicasse melhor também.

Abraços!



Em 12/09/07, Pedro Cardoso[EMAIL PROTECTED] escreveu:
 Douglas,

 muito obrigado pelo link, mas parece que o site simplesmente diz quantas são
 as possibilidades, sem mostrar como o resultado foi obtido. Então, se alguém
 da lista puder expor uma explicação para se chegar ao número correto de
 arranjos de um cubo mágico, ainda agradeço.

 Pedro Lazéra Cardoso

 From: Douglas Ribeiro Silva [EMAIL PROTECTED]
 Reply-To: obm-l@mat.puc-rio.br
 To: obm-l@mat.puc-rio.br
 Subject: Re: [obm-l] Cubo de Rubik
 Date: Tue, 11 Sep 2007 03:19:58 -0300
 
 Ola Pedro!
 
 Sugiro que visite o link http://mathworld.wolfram.com/RubiksCube.html
 que tem varias informações sobre o cubo incluindo o numero de
 combinações possiveis.
 
 Abraços!
 
 Em 11/09/07, Pedro Cardoso[EMAIL PROTECTED] escreveu:
   Olá.
  
   Peço desculpas se alguém já apresentou à lista essa questão - não
 consigo
   achar muita coisa naquele site indicado para pesquisar assuntos
   anteriormente discutidos por aqui. De qualquer forma, aí vai...
  
   Quantas são as possibilidades de arranjo de um cubo mágico, ou cubo de
   rubik? Lembrando que um cubo mágico é um cubo cujas faces são divididas
 em 9
   quadrados iguais, pintados com uma única cor, entre 6 disponíveis. Cada
 cor
   deve ser usada um número idêntico de vezes (9 vezes).
  
   Acredito que seja dispensável, mas, se alguém quiser ver uma imagem:
   http://tatooblue.blogs.sapo.pt/arquivo/cubo_magico.jpg
  
   Queria saber se minha solução esta certa (acho que não).
   Primeiramente, fiz 54!. Depois, dividi por 9! seis vezes, para não
 contar os
   casos em que quadrados de mesma cor foram trocados de posição.
 Finalmente,
   dividi ainda por 24, que é o número de vezes que posso obter
 configurações
   'diferentes' por rotação. Achei um número grande demais - cerca de
 10^38, de
   acordo com a calculadora do windows.
  
   Pedro Lazéra Cardoso

 _
 Inscreva-se no novo Windows Live Mail beta e seja um dos primeiros a testar
 as novidades-grátis. Saiba mais:
 http://www.ideas.live.com/programpage.aspx?versionId=5d21c51a-b161-4314-9b0e-4911fb2b2e6d

 =
 Instruções para entrar na lista, sair da lista e usar a lista em
 http://www.mat.puc-rio.br/~nicolau/olimp/obm-l.html
 =


=
Instruções para entrar na lista, sair da lista e usar a lista em
http://www.mat.puc-rio.br/~nicolau/olimp/obm-l.html
=


Re: [obm-l] Cubo de Rubik

2007-09-11 Por tôpico Douglas Ribeiro Silva
Ola Pedro!

Sugiro que visite o link http://mathworld.wolfram.com/RubiksCube.html
que tem varias informações sobre o cubo incluindo o numero de
combinações possiveis.

Abraços!

Em 11/09/07, Pedro Cardoso[EMAIL PROTECTED] escreveu:
 Olá.

 Peço desculpas se alguém já apresentou à lista essa questão - não consigo
 achar muita coisa naquele site indicado para pesquisar assuntos
 anteriormente discutidos por aqui. De qualquer forma, aí vai...

 Quantas são as possibilidades de arranjo de um cubo mágico, ou cubo de
 rubik? Lembrando que um cubo mágico é um cubo cujas faces são divididas em 9
 quadrados iguais, pintados com uma única cor, entre 6 disponíveis. Cada cor
 deve ser usada um número idêntico de vezes (9 vezes).

 Acredito que seja dispensável, mas, se alguém quiser ver uma imagem:
 http://tatooblue.blogs.sapo.pt/arquivo/cubo_magico.jpg

 Queria saber se minha solução esta certa (acho que não).
 Primeiramente, fiz 54!. Depois, dividi por 9! seis vezes, para não contar os
 casos em que quadrados de mesma cor foram trocados de posição. Finalmente,
 dividi ainda por 24, que é o número de vezes que posso obter configurações
 'diferentes' por rotação. Achei um número grande demais - cerca de 10^38, de
 acordo com a calculadora do windows.

 Pedro Lazéra Cardoso

 _
 Verifique já a segurança do seu PC com o Verificador de Segurança do Windows
 Live OneCare! http://onecare.live.com/site/pt-br/default.htm

 =
 Instruções para entrar na lista, sair da lista e usar a lista em
 http://www.mat.puc-rio.br/~nicolau/olimp/obm-l.html
 =


=
Instruções para entrar na lista, sair da lista e usar a lista em
http://www.mat.puc-rio.br/~nicolau/olimp/obm-l.html
=


[obm-l] Livros de Recorrencia

2007-08-06 Por tôpico Douglas Ribeiro Silva
Alguém tem uma boa indicação online de preferencia ou um bom livro que
trate sobre equações de recorrencia? A única fonte que encontrei foi
um artigo publicado na Eureka:
http://www.obm.org.br/eureka/artigos/recorrencia.pdf

Qualquer ajuda eu agradeço.

Abraços!

Douglas

=
Instruções para entrar na lista, sair da lista e usar a lista em
http://www.mat.puc-rio.br/~nicolau/olimp/obm-l.html
=


Re: [obm-l] ÂNGULO

2007-08-01 Por tôpico Douglas Ribeiro Silva
Olá!

Muito legal esse problema pois ao contrário do que parece, ele possui
2 respostas. Uma para A  90 e outra para A  90. Isso porque muda a
relação do angulo BTC com relação a A.

Para encontrar a resposta use a equação BHC + BIC + BTC = 330.
E escreva os angulos em função de A.
BHC você encontra a partir do quadrilatero inscritivel com diagonal
AH. Os outros vertices são pes das alturas.
BIC use que A + B + C = 180 e BIC = 180 - (B/2 + C/2)
E o BTC é dobro de A, se A  90 e 360 - 2A se A  90.

Substituindo na expressão encontramos duas respostas: 40 ou 120

Abraços!

Douglas.

Em 01/08/07, arkon[EMAIL PROTECTED] escreveu:



 ALGUÉM, POR FAVOR, PODERIA RESOLVER ESSA:



 Num triângulo acutângulo ABC onde H é o ortocentro, I é o incentro e T é o
 circuncentro, a soma dos ângulos BHC, BIC e BTC é 330°. Calcular, em graus,
 o valor do ângulo BAC.



 DESDE JÁ MUITO OBRIGADO

=
Instruções para entrar na lista, sair da lista e usar a lista em
http://www.mat.puc-rio.br/~nicolau/olimp/obm-l.html
=


Re: [obm-l] ÂNGULO

2007-08-01 Por tôpico Douglas Ribeiro Silva
Só fazendo um breve comentário devido a uma falta de atenção minha, o
caso de 120 graus é obviamente para um triangulo obtusangulo(que não é
o que a questão quer).

Abraços

Em 02/08/07, Douglas Ribeiro Silva[EMAIL PROTECTED] escreveu:
 Olá!

 Muito legal esse problema pois ao contrário do que parece, ele possui
 2 respostas. Uma para A  90 e outra para A  90. Isso porque muda a
 relação do angulo BTC com relação a A.

 Para encontrar a resposta use a equação BHC + BIC + BTC = 330.
 E escreva os angulos em função de A.
 BHC você encontra a partir do quadrilatero inscritivel com diagonal
 AH. Os outros vertices são pes das alturas.
 BIC use que A + B + C = 180 e BIC = 180 - (B/2 + C/2)
 E o BTC é dobro de A, se A  90 e 360 - 2A se A  90.

 Substituindo na expressão encontramos duas respostas: 40 ou 120

 Abraços!

 Douglas.

 Em 01/08/07, arkon[EMAIL PROTECTED] escreveu:
 
 
 
  ALGUÉM, POR FAVOR, PODERIA RESOLVER ESSA:
 
 
 
  Num triângulo acutângulo ABC onde H é o ortocentro, I é o incentro e T é o
  circuncentro, a soma dos ângulos BHC, BIC e BTC é 330°. Calcular, em graus,
  o valor do ângulo BAC.
 
 
 
  DESDE JÁ MUITO OBRIGADO


=
Instruções para entrar na lista, sair da lista e usar a lista em
http://www.mat.puc-rio.br/~nicolau/olimp/obm-l.html
=


Re: [obm-l] Uma boa de geometria - CALMA !!!

2007-07-31 Por tôpico Douglas Ribeiro Silva
Olá pessoal!

Muito obrigado pela colaboração de todos na solução do problema.
Enviei a solução para [EMAIL PROTECTED] com as devidas citações ao Nehab e
ao Marcio. Obrigado pela dica da estrategia padrao Marcio!
Certamente será muito útil em problemas futuros.

Por sinal como foi a sua solução para o problema? Fiquei curioso e
creio que outros também estão.

Alguém saberia me dizer se é esse e-mail([EMAIL PROTECTED]) o correto para
enviar as soluções dos problemas propostos da Eureka? Tinha enviado
uma outra vez mas não obtive resposta.

Abraços,

Douglas Ribeiro

OBS: Desculpe a ousadia Nehab, mas foi foi mais forte que eu!


Em 31/07/07, Marcio Cohen[EMAIL PROTECTED] escreveu:
 Douglas,

 Você certamente fez a parte difícil da questão e merece 100% dos créditos
 por isso. Eu tinha feito uma solução por complexos para a questão da Eureka
 na aula de treinamento da imo, mas a sua é muito mais legal!!

 Para provar o detalhe final da sua solução, minha estratégia padrão é:

 Seja a=exp(iA), b=exp(iB), c=exp(iC). Então, abc = -1 e como
 exp(ix)+exp(-ix) = 2cosx:

 (cosA)^2 + (cosB)^2 + (cosC)^2 = (1/4)*(a^2 + 1/a^2 + b^2 + 1/b^2 + c^2 +
 1/c^2 + 6)
 = (1/4)*(a^2 + b^2 + c^2 + (bc)^2 + (ab)^2 + (ac)^2+6);


 8cosA*cosB*cosC = (a+1/a)(b+1/b)(c+1/c) = -(1+a^2)(1+b^2)(1+c^2) =
 -(1+a^2+b^2+c^2+(ab)^2+(ac)^2+(bc)^2+1).

 Substituindo uma na outra, 8cosA*cosB*cosC = -(2+4*( (cosA)^2 + (cosB)^2 +
 (cosC)^2 - 6), ou seja,
 (cosA)^2 + (cosB)^2 + (cosC)^2 = 1 - 2cosAcosBcosC

 Abraços,
 Marcio Cohen


 On 7/30/07, Douglas Ribeiro Silva  [EMAIL PROTECTED] wrote:
 
  Olá Nehab!
 
  Primeiramente gostaria de expressar minha satisfação do problema ter
  de fato chamado sua atenção e do Rogério Ponce. Já participo da
  lista(não muito ativamente) há um bom tempo e percebo que assim como
  eu, vocês gostam muito de geometria.
 
  O problema na verdade veio da minha cabeça, mas foi inspirado em um
  problema proposto na última(ou penúltima) Eureka. Originalmente o
  problema pedia para mostrar que XYZ estão alinhados se e somente se
  cosA*cosB*cosC = -3/8. Então pensei em me inspirar nos chineses, que
  gostavam de resolver teoremas usando áreas, e pensei em zerar a área
  do triangulo XYZ para chegar na tão esperada relação.
 
  Por um lado estou satisfeito, pois consegui chegar em uma expressão
  que relaciona as áreas corretamente, por outro estou frustrado pois
  não consigo fazer a última passagem, que certamente exige uma
  fatoração ou algo do tipo que não estou conseguindo enxergar.
 
  A relação que eu cheguei foi S(XYZ) = S(ABC)*[7 - 4((cosA)^2 +
  (cosB)^2 + (cosC)^2)].
  Note que a relação é válida nos casos mais triviais em que o triangulo
  é equilatero, retangulo(que, como o Rogerio falou, são respectivamente
  1:4 e 1:3) ou isosceles com angulo de 120 graus(basta fazer um desenho
  para ver que a área dá zero). Notem que a relação pedida no problema
  da Eureka é satisfeita para este triangulo isosceles.
 
  Aos curiosos que querem saber como eu cheguei nessa relação, segue a
  idéia abaixo:
 
  Construam o triangulo ABC e suas respectivas reflexões XYZ.
  Observe que S(XYZ) = [S(ABC) + S(BCX) + S(ACY) + S(ABZ)] - S(AYZ) -
  S(XBZ) - S(XYC)
 
  S(ABC) = S(BCX) = S(ACY) = S(ABZ) por construção
 
  As áreas de AYZ XBZ e XYC podem ser somadas ou subtraídas, dependendo
  se os ângulos YAB = 3A, XBZ = 3B ou XCY = 3C forem maiores ou menores
  que 180 graus. Para esses triangulos vou usar que S(AYZ) =
  bc*sen(3A)/2, S(XBZ) = ac*sen(3B)/3 e S(XYC) = ab*sen(3C)/2.
 
  Então a relação passa a ser S(XYZ) = 4S(ABC) - bc*sen(3A)/2 -
  ac*sen(3B)/3 - ab*sen(3C)/2
 
  Agora substituímos sen(3X) = -4*[sen(X)]^3 + 3*sen(X) em todos e
  substituímos também bc/2 ac/2 e ab/2 respectivamente por
  S(ABC)/sen(A), S(ABC)/sen(B) e S(ABC)/sen(C), devido à mesma fórmula
  de área em função dos lados e do angulo para o triangulo original.
 
  Fazendo as devidas substituições acima, simplificamos os senos e
  ficamos com a relação da soma dos quadrados dos senos. Basta trocar
  [sen(X)]^2 por 1 - [cos(X)]^2 e chegamos em S(XYZ) = S(ABC)*[7 -
  4((cosA)^2 + (cosB)^2 + (cosC)^2)].
 
  Quando eu enviei o problema ainda não tinha chegado nesse resultado e
  achava que chegaria em uma expressão mais fácil de passar para o
  produto de cossenos.
 
  Qualquer ajuda para terminar o problema eu agradeço bastante e
  certamente darei os devidos créditos quando enviar a solução para a
  Eureka.
 
  Abraços, Douglas
 
 
 
 
  Em 30/07/07, Carlos Eddy Esaguy Nehab[EMAIL PROTECTED]  escreveu:
  
Oi, querido Ponce
  
Naturalmente não se supunha (pelo menos eu) que a relação entre as
 áreas
   independesse do triângulo, mas mesmo assim, confesso que tentei vários
   caminhos e não encontrei uma solução simples para o problema.
  
Eu esperava algo do tipo:  a razão entre as áreas é o quadrado do
 produto
   dos senos dos angulos, ou  coisa similar.  Embora tendo encontrado
 várias
   coisas curiosas sobre o maldito e interessante

Re: [obm-l] Uma boa de geometria - CALMA !!!

2007-07-30 Por tôpico Douglas Ribeiro Silva
Olá Nehab!

Primeiramente gostaria de expressar minha satisfação do problema ter
de fato chamado sua atenção e do Rogério Ponce. Já participo da
lista(não muito ativamente) há um bom tempo e percebo que assim como
eu, vocês gostam muito de geometria.

O problema na verdade veio da minha cabeça, mas foi inspirado em um
problema proposto na última(ou penúltima) Eureka. Originalmente o
problema pedia para mostrar que XYZ estão alinhados se e somente se
cosA*cosB*cosC = -3/8. Então pensei em me inspirar nos chineses, que
gostavam de resolver teoremas usando áreas, e pensei em zerar a área
do triangulo XYZ para chegar na tão esperada relação.

Por um lado estou satisfeito, pois consegui chegar em uma expressão
que relaciona as áreas corretamente, por outro estou frustrado pois
não consigo fazer a última passagem, que certamente exige uma
fatoração ou algo do tipo que não estou conseguindo enxergar.

A relação que eu cheguei foi S(XYZ) = S(ABC)*[7 - 4((cosA)^2 +
(cosB)^2 + (cosC)^2)].
Note que a relação é válida nos casos mais triviais em que o triangulo
é equilatero, retangulo(que, como o Rogerio falou, são respectivamente
1:4 e 1:3) ou isosceles com angulo de 120 graus(basta fazer um desenho
para ver que a área dá zero). Notem que a relação pedida no problema
da Eureka é satisfeita para este triangulo isosceles.

Aos curiosos que querem saber como eu cheguei nessa relação, segue a
idéia abaixo:

Construam o triangulo ABC e suas respectivas reflexões XYZ.
Observe que S(XYZ) = [S(ABC) + S(BCX) + S(ACY) + S(ABZ)] - S(AYZ) -
S(XBZ) - S(XYC)

S(ABC) = S(BCX) = S(ACY) = S(ABZ) por construção

As áreas de AYZ XBZ e XYC podem ser somadas ou subtraídas, dependendo
se os ângulos YAB = 3A, XBZ = 3B ou XCY = 3C forem maiores ou menores
que 180 graus. Para esses triangulos vou usar que S(AYZ) =
bc*sen(3A)/2, S(XBZ) = ac*sen(3B)/3 e S(XYC) = ab*sen(3C)/2.

Então a relação passa a ser S(XYZ) = 4S(ABC) - bc*sen(3A)/2 -
ac*sen(3B)/3 - ab*sen(3C)/2

Agora substituímos sen(3X) = -4*[sen(X)]^3 + 3*sen(X) em todos e
substituímos também bc/2 ac/2 e ab/2 respectivamente por
S(ABC)/sen(A), S(ABC)/sen(B) e S(ABC)/sen(C), devido à mesma fórmula
de área em função dos lados e do angulo para o triangulo original.

Fazendo as devidas substituições acima, simplificamos os senos e
ficamos com a relação da soma dos quadrados dos senos. Basta trocar
[sen(X)]^2 por 1 - [cos(X)]^2 e chegamos em S(XYZ) = S(ABC)*[7 -
4((cosA)^2 + (cosB)^2 + (cosC)^2)].

Quando eu enviei o problema ainda não tinha chegado nesse resultado e
achava que chegaria em uma expressão mais fácil de passar para o
produto de cossenos.

Qualquer ajuda para terminar o problema eu agradeço bastante e
certamente darei os devidos créditos quando enviar a solução para a
Eureka.

Abraços, Douglas




Em 30/07/07, Carlos Eddy Esaguy Nehab[EMAIL PROTECTED] escreveu:

  Oi, querido Ponce

  Naturalmente não se supunha (pelo menos eu) que a relação entre as áreas
 independesse do triângulo, mas mesmo assim, confesso que tentei vários
 caminhos e não encontrei uma solução simples para o problema.

  Eu esperava algo do tipo:  a razão entre as áreas é o quadrado do produto
 dos senos dos angulos, ou  coisa similar.  Embora tendo encontrado várias
 coisas curiosas sobre o maldito e interessante triângulo, tentando resolver
 o problema, não encontrei nada simples que merecesse ser publicado.

  E também confesso que imaginei que alguém mais inspirado conseguisse alguma
 expressão simples para a resposta.Resta aguardar  que quem propôs o
 problema informe se sabe alguma coisa (aliás hábito pouco praticado em nossa
 lista é informar a origem dos problemas propostos - e às vezes, a origem é
 bastante interessante).

  Eu realmente gosto desta informação pois tenho o hábito (e gosto) de
 mencionar a origem (e a solução) de qualquer problema que eu proponho, no
 mínimo para respeitar a história... e o trabalho alheio.

  Abraços,
  Nehab

  At 01:09 29/7/2007, you wrote:

 Ola' Douglas e colegas da lista,
  nao existe uma relacao fixa entre as 2 areas.

  Num triangulo equilatero a relacao vale 1/4 , e num triangulo retangulo ela
 vale 1/3.
  E repare que podemos girar um dos lados do triangulo equilatero em torno do
 seu ponto medio, de forma a transforma-lo, de forma continua, em triangulo
 retangulo. O efeito disso e' percorrermos todos os valores de 1/4 a 1/3 ,
 por exemplo, mostrando que nao existe uma relacao fixa entre as 2 areas.

  Obviamente poderiamos querer tentar encontrar alguma relacao envolvendo
 outra area notavel (como o triangulo de Euler, por exempo) , alem da area
 dos 2 triangulos originais, mas nao e' o que o problema pede (e nem faria
 muito sentido ficar testando uma infinidade de combinacoes).

  Portanto, a relacao entre as areas ABC e XYZ  e' ... NENHUMA!

  []'s
  Rogerio Ponce


  Douglas Ribeiro Silva [EMAIL PROTECTED] escreveu:
  Seja um triangulo ABC com lados a, b, c.

  X eh a reflexao de A em relacao a reta que passa por BC
  Y eh a reflexao de B em

[obm-l] Uma boa de geometria

2007-07-23 Por tôpico Douglas Ribeiro Silva

Seja um triangulo ABC com lados a, b, c.

X eh a reflexao de A em relacao a reta que passa por BC
Y eh a reflexao de B em relacao a reta que passa por AC
Z eh a reflexao de C em relacao a reta que passa por AB

Qual a relacao entre as areas de ABC e XYZ?
=
Instruções para entrar na lista, sair da lista e usar a lista em
http://www.mat.puc-rio.br/~nicolau/olimp/obm-l.html
=


[obm-l] Problemas da Eureka

2007-07-16 Por tôpico Douglas Ribeiro Silva

Olá pessoal!

Gostaria de confirmar uma coisa... soluções de problemas propostos
pela Eureka devem ser mandados para qual e-mail? Eu enviei para
[EMAIL PROTECTED] mas não tive resposta. Está correto?

Abraços!

=
Instruções para entrar na lista, sair da lista e usar a lista em
http://www.mat.puc-rio.br/~nicolau/olimp/obm-l.html
=


[obm-l] E-mails sobre 0,9999... = 1

2007-05-10 Por tôpico Douglas Ribeiro Silva

Não vou levantar a questão pois já sei que foi largamente discutida
nessa lista, no entanto um amigo me questionou sobre isso essa semana
e queria mostrar a ele o que já foi discutido anteriormente.

Se não me falha a memória houve algum e-mail que tinham vários links e
explicações sobre o fato, no entando não consegui encontra-lo nos
arquivos da lista. Agradeceria se alguém pudesse me passar o conteúdo
da mensagem ou mesmo enviar o link da mesma.

Obrigado pela atenção

Abraços, Douglas

=
Instruções para entrar na lista, sair da lista e usar a lista em
http://www.mat.puc-rio.br/~nicolau/olimp/obm-l.html
=


Re: [obm-l] Re: [obm-l] USAMO - Soma trigonométrica.

2007-03-01 Por tôpico Douglas Ribeiro Silva

Vou aproveitar a soma trigonométrica e pedir novamente uma ajuda com o
produto trigonométrico sen(kPI/n), k indo de 1 até n-1. Sei que o
resultado dá n/2^(n-1) mas não encontrei nenhuma maneira de
demonstrar. Qualquer ajuda eu agradeço.

Abraços.

Douglas

Em 28/02/07, Rogério Possi Júnior[EMAIL PROTECTED] escreveu:

Boa Shine!

Sds,

Rogério


From: Carlos Yuzo Shine [EMAIL PROTECTED]
Reply-To: obm-l@mat.puc-rio.br
To: obm-l@mat.puc-rio.br
Subject: [obm-l] Re: [obm-l] USAMO - Soma trigonométrica.
Date: Wed, 28 Feb 2007 16:44:13 -0800 (PST)

Ah, esse é um grande clássico!

Estamos somando termos da forma 1/(cos k.cos(k+1)), com medidas em graus.

Antes de continuar, vale a pena mostrar um exemplo de soma telescópica
parecida, mas mais simples, que é a soma
1/(1.2) + 1/(2.3) + ... + 1/(88.89)
de termos do tipo 1/(k.(k+1)). A idéia é escrever essa fração como soma de
frações parciais, ou seja, encontrar constantes A e B tais que
1/(k.(k+1)) = A/k + B/(k+1)

Abrindo tudo e fazendo identidade de polinômios, encontramos A = 1 e B =
-1, de modo que a soma é igual a
(1/1 - 1/2) + (1/2 - 1/3) + ... + (1/88 - 1/89) = 1 - 1/89 = 88/89

Tendo essa idéia em vista, vamos encontrar uma função f(n) de Z em R tal
que
1/(cos k.cos(k+1)) = f(k)/cos k - f(k+1)/cos(k+1)

Tirando o mínimo e eliminando denominadores, encontramos
1 = f(k)cos(k+1) - f(k+1)cos k

Parece alguma fórmula familiar? Compare com
sen(a - b) = sen a cos b - sen b cos a
(forçando um pouco mais a barra: faça a = k+1 e b = k)

Então parece valer a pena tomar f(n) = C.sen n. Fazendo umas contas não é
difícil ver que C = -1/sen 1. Assim
1/(cos k.cos(k+1)) = 1/sen1(sen k/cos k - sen(k+1)/cos(k+1))
e a soma pedida é
1/(cos0.cos1) + 1/(cos1.cos2) + ... + 1/(cos88.cos89)
  = -1/sen1((sen0/cos0 - sen1/cos1) + (sen1/cos1 - sen2/cos2) + ... +
(sen88/cos88 - sen89/cos89))
  = -1/sen1(sen0/cos0 - sen89/cos89)
  = -1/sen1(0 - cos1/sen1)
  = cos1/sen^2(1).

[]'s
Shine


- Original Message 
From: Rogério Possi Júnior [EMAIL PROTECTED]
To: obm-l@mat.puc-rio.br
Sent: Wednesday, February 28, 2007 8:11:51 PM
Subject: [obm-l] USAMO - Soma trigonométrica.


Caros,

Alguém consegue resolver essa usando soma telescópica?

(USAMO-1992) Mostre que 1/(cos 0.cos 1) + 1/(cos 1.cos2) + ... +
1/(cos88.cos89) = cos(1)/sen^2(1).

Rogério

_
MSN Messenger: instale grátis e converse com seus amigos.
http://messenger.msn.com.br

=
Instruções para entrar na lista, sair da lista e usar a lista em
http://www.mat.puc-rio.br/~nicolau/olimp/obm-l.html
=




Sucker-punch spam with award-winning protection.
Try the free Yahoo! Mail Beta.
http://advision.webevents.yahoo.com/mailbeta/features_spam.html

=
Instruções para entrar na lista, sair da lista e usar a lista em
http://www.mat.puc-rio.br/~nicolau/olimp/obm-l.html
=

_
MSN Messenger: instale grátis e converse com seus amigos.
http://messenger.msn.com.br

=
Instruções para entrar na lista, sair da lista e usar a lista em
http://www.mat.puc-rio.br/~nicolau/olimp/obm-l.html
=



=
Instruções para entrar na lista, sair da lista e usar a lista em
http://www.mat.puc-rio.br/~nicolau/olimp/obm-l.html
=


Re: [obm-l] Teorema - CARAMBA, qual deles, então???

2007-02-13 Por tôpico Douglas Ribeiro Silva

Olá Nehab!

Acho que o forum teorema que o Eduardo se refere é um forum que
existiu até dezembro(eu acho) do ano passado em que havia uma
quantidade razoável de pessoas discutindo problemas e outros assuntos
relacionados a matemática. Por sinal, neste mesmo forum, foi publicada
uma nota de falecimento do professor Morgado na época do infortúnio. O
forum também contava com uma seção para discussão de física e uma
parte específica para postar reviews de livros ligados a matemática.
Apesar de se tratar de um forum muito bom, o mesmo encerrou suas
atividades não sei porquê. Se alguem da lista pudesse esclarecer
ficaria bastante grato.

Abraços,

Douglas Ribeiro Silva

Em 13/02/07, Eduardo Wilner[EMAIL PROTECTED] escreveu:

Haver ? Deixa prá lá...

Mas o grupoteorema não  tem nada a ver com o Foruim Teorema.

Carlos Eddy Esaguy Nehab [EMAIL PROTECTED] escreveu:

Ué,

Não entendí =-O!  Explica para mim, please :-(    Será que está
ocorrendo um choque linguístico de gerações? (hoje vocês falam tô chegando e
tão saindo;  sinistro virou superlativo de qualquer coisa etc, etc...
durma-se com um barulho destes!).   Mas não tem nada não...

Paguei algum mico e não percebi?  Não era http://www.grupoteorema.mat.br/
que você queria?  O que era outro teorema?  O de Pitágoras? Ou o mico foi
seu?   E olha (agora eu me vingo)  não tem nada a haver... ou  não
tem nada a ver ???

Abraços  :-) ,
Nehab

At 21:56 12/2/2007, you wrote:

Como dizem hoje em dia:
  Não tem nada a ver.

Carlos Eddy Esaguy Nehab [EMAIL PROTECTED] escreveu:
 Oi, Max,

 Já mudou há um bom tempo para http://www.grupoteorema.mat.br/

 Abraços,
 Nehab

 At 23:18 11/2/2007, you wrote:


 Por que o site do Teorema saiu do ar ?

 
 O Windows Live Spaces está aqui! Descubra como é fácil criar seu espaço na
Web e sua rede amigos. Confira!

__
Fale com seus amigos de graça com o novo Yahoo! Messenger
http://br.messenger.yahoo.com/


 __
Fale com seus amigos de graça com o novo Yahoo! Messenger
http://br.messenger.yahoo.com/


=
Instruções para entrar na lista, sair da lista e usar a lista em
http://www.mat.puc-rio.br/~nicolau/olimp/obm-l.html
=


Re: [obm-l] Agradecimentos!!!

2007-02-11 Por tôpico Douglas Ribeiro Silva

Se é sobre o problema que eu estou pensando eu ficaria bastante grato
se pudessem disponibilizar o link para todos =)

Abraços

Em 09/02/07, Nicolau C. Saldanha[EMAIL PROTECTED] escreveu:

On Thu, Feb 08, 2007 at 09:59:44PM -0200, cfgauss77 wrote:
   Valeu Nicolau, para aqueles que quiserem baixar o arquivo ficou legal!!!

Mas acho que só nós dois sabemos onde baixar o arquivo... :-)

[]s, N.
=
Instruções para entrar na lista, sair da lista e usar a lista em
http://www.mat.puc-rio.br/~nicolau/olimp/obm-l.html
=



=
Instruções para entrar na lista, sair da lista e usar a lista em
http://www.mat.puc-rio.br/~nicolau/olimp/obm-l.html
=


[obm-l] Poligono de n lados + Produtorio Trigonometrico

2007-02-06 Por tôpico Douglas Ribeiro Silva

Olá a todos!

Gostaria de uma ajuda num problema tirado do maravilhoso livro
Geometria II do Wagner, Morgado e Jorge: Sejam P1, P2, ... , Pn os
vertices de um polígono regular de n lados inscrito em um círculo de
raio 1. Mostre que (P1P2)*(P1P3)*(P1P4)*...*(P1Pn) = n
Após alguns cálculos eu resolvi o problema, no entando estou
dependendo de provar a identidade do produtório trigonométrico a
seguir: 
http://functions.wolfram.com/ElementaryFunctions/Sin/24/01/0001/MainEq1.L.gif

Alguém teria uma maneira simples de demonstrar essa identidade ou tem
uma boa idéia pra resolver o problema sem usá-la?

Abraços,

Douglas Ribeiro

=
Instruções para entrar na lista, sair da lista e usar a lista em
http://www.mat.puc-rio.br/~nicolau/olimp/obm-l.html
=


Re: [obm-l] Poligono de n lados + Produtorio Trigonometrico

2007-02-06 Por tôpico Douglas Ribeiro Silva

Olá! Acho que seu arquivo não chegou... (Ou foi só comigo que isso aconteceu?)
Poderia manda-lo novamente?
Obrigado!

Em 06/02/07, cfgauss77[EMAIL PROTECTED] escreveu:

 Vai em anexo um arquivo em PDF que pode te ajudar!!!
   abraços!!!



Olá a todos!

 Gostaria de uma ajuda num problema tirado do maravilhoso livro
 Geometria II do Wagner, Morgado e Jorge: Sejam P1, P2, ... , Pn os
 vertices de um polígono regular de n lados inscrito em um círculo de
 raio 1. Mostre que (P1P2)*(P1P3)*(P1P4)*...*(P1Pn) = n
 Após alguns cálculos eu resolvi o problema, no entando estou
 dependendo de provar a identidade do produtório trigonométrico a
 seguir:
http://functions.wolfram.com/ElementaryFunctions/Sin/24/01/0001/MainEq1.L.gif

 Alguém teria uma maneira simples de demonstrar essa identidade ou tem
 uma boa idéia pra resolver o problema sem usá-la?

 Abraços,

 Douglas Ribeiro


=
 Instruções para entrar na lista, sair da lista e usar a lista em
 http://www.mat.puc-rio.br/~nicolau/olimp/obm-l.html

=


=
Instruções para entrar na lista, sair da lista e usar a lista em
http://www.mat.puc-rio.br/~nicolau/olimp/obm-l.html
=


Re: [obm-l] Livro

2007-01-31 Por tôpico Douglas Ribeiro Silva

Olá!

Bom, até onde eu sei o vol 11 da coleção so foi lançado na ultima
edição da mesma, coisa de no maximo 2 ou 3 anos atras, então não sei
se da pra encontra-lo em sebos com preço bom. Mas se preço não for
problema, faça uma busca no buscape bondfaro ou algum outro site de
busca de produtos. Abaixo segue um link.

http://compare.buscape.com.br/prod_unico?idu=1853570462pos=1site_origem=11642

Abraços



Em 31/01/07, Itamar Sales[EMAIL PROTECTED] escreveu:

Desculpem-me por não ter posto off topic.


From: Itamar Sales [EMAIL PROTECTED]
Reply-To: obm-l@mat.puc-rio.br
To: obm-l@mat.puc-rio.br
Subject: [obm-l] Livro
Date: Wed, 31 Jan 2007 14:04:42 +

Ei pessoal, vocês sabem me dizer se o livro Fundamentos de Matemática
Elementar volume 11, edição com novos testes de vestibulares, já tem pra
vender? E caso tenha, aonde?
Ah, mais uma coisa: O manual do professor não é comercializado, como já vi,
mas de que maneira eu posso conseguí-lo?

Peço desculpas se o assunto desse tópico é repetido, pois não me recordo de
nenhum outro.

Grato. :)

_
Descubra como mandar Torpedos do Messenger para o celular!
http://mobile.msn.com/

=
Instruções para entrar na lista, sair da lista e usar a lista em
http://www.mat.puc-rio.br/~nicolau/olimp/obm-l.html
=

_
Insta-le já o Windows Live Messenger. A nova geração do messenger.
http://get.live.com/messenger/overview

=
Instruções para entrar na lista, sair da lista e usar a lista em
http://www.mat.puc-rio.br/~nicolau/olimp/obm-l.html
=



=
Instruções para entrar na lista, sair da lista e usar a lista em
http://www.mat.puc-rio.br/~nicolau/olimp/obm-l.html
=


Re: [obm-l] Re: [obm-l] Questão de geometria morgado

2006-11-23 Por tôpico Douglas Ribeiro Silva

Murilo, vc nao pode afirmar que DGCE eh um losangulo por suas
diagonais cruzarem em 90 graus. So e se as diagonais se cruzarem em 90
graus e no seu ponto medio.

Emanuel, GDC = 40, pois ADE = 80 e CDE = 60. Então DGI = 50
DI = IH
Entao pelo criterio Lado, Angulo, Lado(GI, Angulo Reto, IH), GIH é
semelhante a GID. Logo GHD = 40.

Abraços

Em 22/11/06, Murilo RFL[EMAIL PROTECTED] escreveu:

Desculpe pela questao errada provavelmente estava de ressaca rsrs.

bom, como DGCE eh um losangulo por suas diagonais cruzarem em 90 graus,
conluimos q DCG == DCE == y

no triangulo ABC isocele eh facil ver q os angulos medem 20 80 80

logo DBE = 20 graus

como DE // BC, DEB == 60 graus

no triangulo DEB possui angulos 20 60 e BDE == 100 graus

como DGEC eh um losangulo o angulo CDE == GDC == 50 (soma = 100 == GDE)

DEC == 100 pois DEA == 80 (triangulo semelhante a ABC)

logo y = 30 pois esta no triangulo de angulos 100 50 30

bom, finalmente 2y + x = 80

x = 20

Desculpe as más explicaçoes de antes e desculpe o erro cometido

Abraços!


- Original Message -
From: claudio.buffara [EMAIL PROTECTED]
To: obm-l obm-l@mat.puc-rio.br
Sent: Thursday, November 16, 2006 9:38 AM
Subject: [obm-l] Re:[obm-l] Re: [obm-l] Questão de geometria morgado


 -- Cabeçalho original ---

 De: [EMAIL PROTECTED]
 Para: obm-l@mat.puc-rio.br
 Cópia:
 Data: Thu, 16 Nov 2006 06:18:46 -0200
 Assunto: [obm-l] Re: [obm-l] Questão de geometria morgado

 Como DC corta o segmento GE em 90º

 concluimos q DCG == DCE == y


 Como voce conclui isso? Isso soh serah verdade se CEG for isosceles, mas
 voce nao provou que eh.


 2y + x = 80

 EDG == EBC == 60

 y + 60 + 100 = 180 no triangulo DEC

 y = 20

 x= 40
   - Original Message -
   From: mentebrilhante brilhante
   To: obm-l@mat.puc-rio.br
   Sent: Thursday, November 16, 2006 12:58 AM
   Subject: [obm-l] Questão de geometria morgado


   Quem puder  ajuda  agradeço


 http://img127.imagevenue.com/img.php?image=47375_geometria_122_568lo.jpg




 =
 Instruções para entrar na lista, sair da lista e usar a lista em
 http://www.mat.puc-rio.br/~nicolau/olimp/obm-l.html
 =


=
Instruções para entrar na lista, sair da lista e usar a lista em
http://www.mat.puc-rio.br/~nicolau/olimp/obm-l.html
=



=
Instruções para entrar na lista, sair da lista e usar a lista em
http://www.mat.puc-rio.br/~nicolau/olimp/obm-l.html
=


Re: [obm-l] Re:[obm-l] Re: [obm-l] Questão de geometria morgado

2006-11-16 Por tôpico Douglas Ribeiro Silva

Usando a figura que voce postou, chame o pto de intersecao de CD com EB de H.

Trace o segmento GH. Note que GHDE é um quadrilatero formado por 2
triangulos isosceles de mesma base DH. Com algumas contas chega-se que
DEG = 30, GEH = 30, GDH = 40, GHD = 40. Depois disso, BHC = 60, GHB =
HEG + EGH(Teorema do angulo externo) = 30 + 50 = 80.O triangulo BGH é
isosceles (20, 80, 80). GB = HB = BC. GBC é isosceles. GCB = 50

Acho que é isso. Se Tiver algum erro por favor avisem

Abraços, Douglas



Em 16/11/06, claudio.buffara[EMAIL PROTECTED] escreveu:

-- Cabeçalho original ---

De: [EMAIL PROTECTED]
Para: obm-l@mat.puc-rio.br
Cópia:
Data: Thu, 16 Nov 2006 06:18:46 -0200
Assunto: [obm-l] Re: [obm-l] Questão de geometria morgado

 Como DC corta o segmento GE em 90º

 concluimos q DCG == DCE == y


Como voce conclui isso? Isso soh serah verdade se CEG for isosceles, mas voce 
nao provou que eh.


 2y + x = 80

 EDG == EBC == 60

 y + 60 + 100 = 180 no triangulo DEC

 y = 20

 x= 40
   - Original Message -
   From: mentebrilhante brilhante
   To: obm-l@mat.puc-rio.br
   Sent: Thursday, November 16, 2006 12:58 AM
   Subject: [obm-l] Questão de geometria morgado


   Quem puder  ajuda  agradeço

   http://img127.imagevenue.com/img.php?image=47375_geometria_122_568lo.jpg




=
Instruções para entrar na lista, sair da lista e usar a lista em
http://www.mat.puc-rio.br/~nicolau/olimp/obm-l.html
=



=
Instruções para entrar na lista, sair da lista e usar a lista em
http://www.mat.puc-rio.br/~nicolau/olimp/obm-l.html
=


Re: [obm-l] Reta de Simpson

2006-10-19 Por tôpico Douglas Ribeiro Silva

http://www.bibvirt.futuro.usp.br/textos/hemeroteca/rpm/rpm44/rpm44_02.pdf

Da uma olhada aí Marinho. Espero que ajude.

Abraços!

Em 19/10/06, Marinho Kamiroski[EMAIL PROTECTED] escreveu:

Na demonstração que os tres pontos desta reta são colineares (hum...), os
livros dizem que existe dois quadrilateros inscritiveis, e certos angulos
iguais. Se alguem souber estas passagens, poderia me explica-las.

_
Inscreva-se no novo Windows Live Mail beta e seja um dos primeiros a testar
as novidades-grátis. Saiba mais:
http://www.ideas.live.com/programpage.aspx?versionId=5d21c51a-b161-4314-9b0e-4911fb2b2e6d

=
Instruções para entrar na lista, sair da lista e usar a lista em
http://www.mat.puc-rio.br/~nicolau/olimp/obm-l.html
=



=
Instruções para entrar na lista, sair da lista e usar a lista em
http://www.mat.puc-rio.br/~nicolau/olimp/obm-l.html
=


[obm-l] Problema de Steiner?!

2006-09-14 Por tôpico Douglas Ribeiro Silva

Dadas n retas não coincidentes num plano, tal que 3 retas não
concorram no mesmo ponto, determinar o número de pontos de intersecção
e o número de regiões em que é dividido o plano.

Um professor me falou que esse problema e outros desse tipo foi
estudado por Steiner, no entanto tentei procuprar algo sobre isso na
internet e não encontrei. Alguém pode confirma isso e/ou mandar alguma
referência sobre problemas de combinatória envolvendo geometria? Sei
resolver usando PA de ordem superior mas gostaria de outras sugestões
de solução.

Abraços, Douglas.

=
Instruções para entrar na lista, sair da lista e usar a lista em
http://www.mat.puc-rio.br/~nicolau/olimp/obm-l.html
=


[obm-l] Somatorios e Produtorios Trigonometricos

2006-09-13 Por tôpico Douglas Ribeiro Silva

Alguém por acaso teria um link da internet com referência de como
resolver somatórios e produtórios trigonométricos?

Abraços!

=
Instruções para entrar na lista, sair da lista e usar a lista em
http://www.mat.puc-rio.br/~nicolau/olimp/obm-l.html
=


Re: [obm-l] Livro

2006-08-22 Por tôpico Douglas Ribeiro Silva

Pois é... aparentemente a dover publications não edita mais o livro,
mas valeu pelas dicas Antonio!

Em 22/08/06, Antonio Neto[EMAIL PROTECTED] escreveu:

Tenho tres solucoes:

1 - Na www.doverpublications.com

2 - Se vc for do Rio de Janeiro, a Interciencia traz para cá, talvez para a
sua cidade, cobrando o frete, é claro. Av Pres. Vargas, 435, 18 andar,
(0xx21) 22 21 09 93.

3 - A MF importa tambem, mas fechou a loja do Rio, mas se vc for de SP eh a
melhor pedida.

   Soh tenho uma pergunta, este livro ainda estah sendo editado? O melhor
seria entrar na Dover e verificar, se nao quiser comprar com eles anota pelo
menos o ISBN, que facilita a busca das livrarias especializadas. Abraco,
olavo.


From: Douglas Ribeiro Silva [EMAIL PROTECTED]
Reply-To: obm-l@mat.puc-rio.br
To: obm-l@mat.puc-rio.br
Subject: [obm-l] Livro
Date: Mon, 21 Aug 2006 22:09:04 -0300

Alguem sabe onde eu posso adquirir o livro Advanced Euclidean
Geometry de R.A. Johnson, Dover Publications, 1960 ?

_
MSN Messenger: instale grátis e converse com seus amigos.
http://messenger.msn.com.br

=
Instruções para entrar na lista, sair da lista e usar a lista em
http://www.mat.puc-rio.br/~nicolau/olimp/obm-l.html
=



=
Instruções para entrar na lista, sair da lista e usar a lista em
http://www.mat.puc-rio.br/~nicolau/olimp/obm-l.html
=


[obm-l] Livro

2006-08-21 Por tôpico Douglas Ribeiro Silva

Alguem sabe onde eu posso adquirir o livro Advanced Euclidean
Geometry de R.A. Johnson, Dover Publications, 1960 ?
=
Instruções para entrar na lista, sair da lista e usar a lista em
http://www.mat.puc-rio.br/~nicolau/olimp/obm-l.html
=


[obm-l] Software para criação de Poliedros

2004-12-01 Por tôpico Douglas Ribeiro Silva








Olá pessoal! Algum de vocês aqui da lista sabe de algum
software que possa fazer criação de poliedros?

Ou então algum software que tenha vários poliedros nele e o próprio
software nos forneça a proporção dos seus lados?

No caso dessa segunda pergunta gostaria de um que preferenciamente
que trabalhasse com poliedros estrelados também.



Qualquer ajuda eu agradeço =)



Abraços, Douglas








[obm-l] RES: [obm-l] Dinâmica

2004-11-02 Por tôpico Douglas Ribeiro Silva








Usando F*dt = m*dv



F*4 = m*20

(F/2)*8 = 2m*v



Dividindo-se uma pela outra, v = 10m/s











De: owner-[EMAIL PROTECTED] [mailto:owner-[EMAIL PROTECTED]] Em nome de Daniela Yoshikawa
Enviada em: terça-feira, 2 de
novembro de 2004 DouGz 13:53
Para: [EMAIL PROTECTED]
Assunto: [obm-l] Dinâmica







Olá!











Vai aí uma de física.











-Uma força de intensidade F, aplicada em um corpo de massa m que se
encontra inicialmente em repouso, é capaz de conferir-lhe velocidade de 20 m/s
em apenas 4s. Dobrando a massa do corpo e reduzindo a força à metade, qual a
intensidade da velocidade adquirida pelo corpo em 8s?











Obrigada.









Yahoo!
Acesso Grátis - Internet rápida e grátis. Instale o discador agora!








[obm-l] Número Phi/Secção Áurea

2004-11-02 Por tôpico Douglas Ribeiro Silva








Alguém sabe uma página na internet com boas informações cotidianas
sobre o número Phi e/ou Secção áurea

Por exemplo onde podemos encontra-lo no cotidiano e etc?








[obm-l] Eixo Radical

2004-10-06 Por tôpico Douglas Ribeiro Silva








Alguém sabe como determinar o eixo radical de 2 circunferências
que não possuem pontos em comum usando régua e compasso?








[obm-l] Jornais

2004-10-06 Por tôpico Douglas Ribeiro Silva








Uma certa banca de revistas vende os jornais A e B. O custo
por jornal é R$1,20 e R$0,40, e ela os vende por R$2,00 e R$0,80
respectivamente. Determine o número de jornais A que a banca deve comprar de
modo a maximizar o seu lucro, sabendo que ela dispõe de R$1999,20, e que a
produção dos jornais não ultrapassa 1500 e 3000 respectivamente.








[obm-l] RES: [obm-l] Re:[obm-l] Re: [obm-l] RE: [obm-l] História da Matemática

2004-09-28 Por tôpico Douglas Ribeiro Silva
Somos 5!
[EMAIL PROTECTED]

Que tal alguém disponibilizar o arquivo num site online? =)

-Mensagem original-
De: [EMAIL PROTECTED] [mailto:[EMAIL PROTECTED] Em nome
de brunno184
Enviada em: terça-feira, 28 de setembro de 2004 DouGz 16:27
Para: obm-l
Assunto: [obm-l] Re:[obm-l] Re: [obm-l] RE: [obm-l] História da Matemática

Eu gostaria
[EMAIL PROTECTED]
obrigado

 Início da mensagem original ---

  De: [EMAIL PROTECTED]
Para: [EMAIL PROTECTED]
  Cc: 
Data: Mon, 27 Sep 2004 17:22:09 -0300
 Assunto: [obm-l] Re: [obm-l] RE: [obm-l] História da 
Matemática

 MensagemEu gostaria
 [EMAIL PROTECTED]
 Grato
 
 Daniel
   - Original Message - 
   From: Leandro Lacorte Recova 
   To: [EMAIL PROTECTED] 
   Sent: Monday, September 27, 2004 11:43 AM
   Subject: [obm-l] RE: [obm-l] História da Matemática
 
 
   Eu conheco um artigo em PDF do Manfredo Carmo sobre 
a Historia da Geometria Diferencial no Brasil. 
 
 
 
   Me avise se quiser pois posso te mandar !
 
 
 
   Leandro
 
 
 
   -Original Message-
   From: [EMAIL PROTECTED] [mailto:owner-obm-
[EMAIL PROTECTED] On Behalf Of Cloves Jr
   Sent: Monday, September 27, 2004 5:51 AM
   To: Grupo OBM
   Subject: [obm-l] História da Matemática
 
 
 
   Olá pessoal...
 
 
 
   Eu sei que o assunto é um pouco off-topic mas 
gostaria de saber se alguém sabe alguma referência que 
eu poderia consultar sobre um trabalho sobre a História 
da Maremática no Brasil...
 
 
 
   Qualquer referência já seria de grande ajuda...
 
 
 
   []s
 
 
 
   Cloves Jr
 
 
 
   ---
   Os e-mails enviados são certificados como livres de 
vírus.
   Checked by AVG anti-virus system 
(http://www.grisoft.com).
   Version: 6.0.768 / Virus Database: 515 - Release 
Date: 22/09/04
 
 
 
 
__
Acabe com aquelas janelinhas que pulam na sua tela.
AntiPop-up UOL - É grátis!
http://antipopup.uol.com.br/



=
Instruções para entrar na lista, sair da lista e usar a lista em
http://www.mat.puc-rio.br/~nicolau/olimp/obm-l.html
=


=
Instruções para entrar na lista, sair da lista e usar a lista em
http://www.mat.puc-rio.br/~nicolau/olimp/obm-l.html
=


[obm-l] Analítica, Plana e Trigonometria

2004-09-16 Por tôpico Douglas Ribeiro Silva








Olá pessoal. Gostaria de uma ajuda nas seguintes questões:



1ª   Achar a equação da reta que passa pelo ponto de
intersecção de duas retas: 11x +3y -7 = 0 e 12x +y -10 = 0 e que se encontra a
igual distancia dos pontos A(3 , -2) e B(-1 , 6). A questão deverá ser resolvida
sem calcular as coordenadas do ponto de intersecção dessas retas.



2ª  Determinar e identificar a equação do lugar geométrico
do centro de uma circunferência que é sempre tangente às circunferências x^2 +
y^2 -4y -12 = 0 e x^2 + y^2 = 1



3ª  Calcular o volume de uma pirâmide regular de
altura h, sabendo-se que a base dessa pirâmide é um polígono regular cuja soma
dos ângulos internos é igual a n*Pi, e a razão entre a área da base e a área
lateral é igual a k.



4ª  Eliminar o arco x
entre as equações:



sen(x)/a = sen(3x)/b = sen(5x)/c



Qualquer ajuda eu agradeço =)








[obm-l] Desigualdade de Médias

2004-09-02 Por tôpico Douglas Ribeiro Silva








Olá pessoal.



Ultimamente eu me deparei com uma questão de média aritmética
x geométrica e fiquei curioso pra saber a generalização da desigualdade da
mesma. Dei uma olhada no arquivo da lista e achei esse link onde o Morgado
mostrou: http://www.mat.puc-rio.br/~nicolau/olimp/obm-l.27/msg00188.html

Entendi o modo como foi feito para quantidade de números potencias
de 2, mas não compreendi os passos utilizados na indução. Alguém poderia
detalhar melhor os passos da demonstração e/ou mandar outras demonstrações
dessa generalização?



Um abraço, Douglas








[obm-l] RES: [obm-l] GEOMETRIA E IMAGINAÇÃO!

2004-08-09 Por tôpico Douglas Ribeiro Silva
Marcos, acho que você se equivocou na resposta.

Realmente Jorge, o problema é interessantíssimo e nunca tinha me deparado
com algo similar.

Se fizerem um esboço do poliedro resultante vão ver que existe a junção de
dois ângulos poliédricos.
Fazendo os pontos A=G, D=I, E=H.

Vamos encontrar o valor destes ângulos.

No Tetraedro:
l^2 = 2(l*sqrt(3)/2)^2 - 2[(l*sqrt(3)/2)^2]* cos(T)
Resolvendo temos que cos(T) = 1/3

Na Pirâmide:
[L*sqrt(2)]^2 = 2(l*sqrt(3)/2)^2 - 2[(l*sqrt(3)/2)^2]* cos(P)
Resolvendo temos que cos(P) = -1/3

Com isso descobrimos que estes ângulos são suplementares, e a junção deles
forma um plano perfeito!

Isto ocorre com a junção de duas faces do tetraedro com a pirâmide, e o
sólido resultante possui duas faces triangulares e três faces
quadrangulares.

Belíssima questão de Geometria Jorge! Se tiver mais dessas mande! =)

Um abraço, Douglas Ribeiro Silva


-Mensagem original-
De: [EMAIL PROTECTED] [mailto:[EMAIL PROTECTED] Em nome
de Marcos Paulo
Enviada em: segunda-feira, 9 de agosto de 2004 20:50
Para: [EMAIL PROTECTED]
Assunto: Re: [obm-l] GEOMETRIA E IMAGINAÇÃO!





=
De:[EMAIL PROTECTED]
Para:[EMAIL PROTECTED]
Assunto:[obm-l] GEOMETRIA E IMAGINAÇÃO!

PASMEM! Este problema de geometria, proposto
numa prova para mais de um milhão
de alunos, teve somente um único acertador,
Daniel Lowen, de 17 anos da Escola
Cocoa Beach

Seja ABCDE uma pirâmide de base quadrada, cujas
faces laterais são triângulos
equiláteros; e seja FGHI um tetaedro regular
cujas faces sejam (triângulos
equiláteros) congruentes às faces laterais da
pirâmide. Suponhamos que se
juntem os sólidos de maneira que a face ADE da
pirâmide coincida com a face GIH
do tetaedro, o resultado sendo o poliedro
ABCDEF. Quantas faces tem este
poliedro?
Há Uma face quadrangular e 6 faces trîangulares.



 
   
   (Educational Testing Service-EUA)

NOTA: Meus amigos, sem nenhum exagero, este é um
problema fascinante. Nunca vi
nada igual. (CAMPEÃO!).



__
WebMail UNIFOR - http://www.unifor.br.

=
Instruções para entrar na lista, sair da lista e
usar a lista em
http://www.mat.puc-rio.br/~nicolau/olimp/obm-l.h
tml

=







=
Instruções para entrar na lista, sair da lista e usar a lista em
http://www.mat.puc-rio.br/~nicolau/olimp/obm-l.html
=


=
Instruções para entrar na lista, sair da lista e usar a lista em
http://www.mat.puc-rio.br/~nicolau/olimp/obm-l.html
=


[obm-l] Lado do Quadrado

2004-07-08 Por tôpico Douglas Ribeiro Silva








Olá pessoal

Qualquer ajuda pra calcular o lado desse quadrado eu fico
grato



[]s






attachment: quadrado.gif

[obm-l] RES: [obm-l] RE: [obm-l] Números Interessantes

2004-05-29 Por tôpico Douglas Ribeiro Silva
Olá Rogério, eu dei uma olhada na sua resolução para o problema e não
discuto que a matemática que você usou está correta... Mas acho que a
intenção do problema proposto não foi chegar na resposta que você chegou.

A questão pede Quantos são os números com 10 algarismo diferentes entre
si e divisível por 1

Você achou todos os números inteiros divisíveis por 1 que tem 10
algarismos, mas não necessariamente algarismos distintos.

Por exemplo: 10 possui 5 zeros e 5 uns

O numero teria que ser do tipo 1896753042
Ao meu ver foi isso que a questão pediu...

Um abraço, Douglas Ribeiro

-Mensagem original-
De: [EMAIL PROTECTED] [mailto:[EMAIL PROTECTED] Em nome
de Rogério Moraes de Carvalho
Enviada em: sexta-feira, 28 de maio de 2004 06:48
Para: [EMAIL PROTECTED]
Assunto: [obm-l] RE: [obm-l] Números Interessantes

Olá André,

Eu já havia postado uma resolução possível para este problema. Veja
o link: http://www.mat.puc-rio.br/~nicolau/olimp/obm-l.200405/msg00793.html.

Atenciosamente,

Rogério Moraes de Carvalho

From: [EMAIL PROTECTED] [mailto:[EMAIL PROTECTED] On
Behalf Of Andre
Sent: quinta-feira, 27 de maio de 2004 23:59
To: [EMAIL PROTECTED]
Subject: [obm-l] Números Interessantes

Olá,
 
    Questão:  Quantos são os números com 10 algarismo diferentes entre
si e divisível por 1.
    Dizer que eles estão incluídos entre os números interessantes está
correto?



=
Instruções para entrar na lista, sair da lista e usar a lista em
http://www.mat.puc-rio.br/~nicolau/olimp/obm-l.html
=


=
Instruções para entrar na lista, sair da lista e usar a lista em
http://www.mat.puc-rio.br/~nicolau/olimp/obm-l.html
=


[obm-l] RES: [obm-l] Simplificação Trigonométria

2004-04-05 Por tôpico Douglas Ribeiro Silva
s80/(2c20-c80)

Fazendo que s80 = s(50 + 30) e s20 = s(50 - 30)

(S50c30 + c50s30)/[2(c50c30 + s50s30) - c50c30 + s50s30]

desenvolvendo o produto no denominador e agrupando os termos fica:

(s50c30 + s30c50)/(c50c30 + 3s50s30)

substituindo os valores de s30 e c30:

(s50sqrt(3)/2 + c50/2)/(c50sqrt(3)/2 + 3s50/2)

(s50sqrt(3) + c50)/(c50sqrt(3) + 3s50)

evidenciando sqrt(3) no numerador e 3 no denominador:

sqrt(3)[s50 + c50/sqrt(3)]/3[s50 + c50/sqrt(3)]

simplificando:

sqrt(3)/3 = tg30°


Elegante ou não, o problema foi resolvido =). Agradeço bastante ao
Ricardo(e ao Rafael Sampaio também) por terem se dado o trabalho de me
ajudar nessa questão.

Recebi agora a pouco um e-mail do Rafael dizendo que conseguiu
solucionar o problema também. Não sei qual a forma que ele encontrou mas
estou bastante curioso para ver.

Muito obrigado mesmo pessoal por estarem me dando essa mãozona!

Um abraço, Douglas Ribeiro Silva


-Mensagem original-
De: [EMAIL PROTECTED] [mailto:[EMAIL PROTECTED] Em
nome de Ricardo Bittencourt
Enviada em: segunda-feira, 5 de abril de 2004 00:38
Para: [EMAIL PROTECTED]
Assunto: Re: [obm-l] Simplificação Trigonométria

Douglas Ribeiro Silva wrote:

 Olá pessoal... estou com um problema para simplificar uma expressão
 trigonométrica mas não consegui encontrar relações para chegar na
 simplificação. A expressão é:
 tg²(80°) * [tg(60°) – tg(50°)] / tg²(80°) – [tg(50°) * tg(60°)]

Aqui vai uma solução incompleta e totalmente
deselegante. Eu mandei o resultado pacrial pro Douglas e
ele matou o resto, então coloco aqui a minha parte e
depois ele completa com a parte dele!

Vou usar uma notação simplificada, e, espero, intuitiva.


...primeiro, substituí todos os tg por sen/cos...

(t80t80*(t60-t50))/(t80t80-t50t60)=

((s80s80/c80c80)*(s60/c60-s50/c50))/((s80s80/c80c80)-(s60s50/c60c50))=

...distribuí o denominador e cortei c80c80c60c50 da fração...

(s80s80(s60c60-s50c60))/(s80s80c60c50-c80c80s60s50)=

...no numerador, s60c60-s50c60 = s10...

(s80s80s10)/(s80s80c60c50-c80c80s60s50)=

...no denominador, s80s80=(1-c80c80)...

(s80s80s10)/((1-c80c80)*c60c50-c80c80s60s50)=
(s80s80s10)/(c60c50-c80c80c60c50-c80c80s60s50)=
(s80s80s10)/(c60c50-c80c80(c60c50-s60s50))=

...c60c50-s60s50 = c10...

(s80s80s10)/(c60c50-c80c80c10)=

...agora, c60c50 = (c110+c10)/2...

(s80s80s10)/(((c110+c10)/2)-c80c80c10)=

...c110=s(90-110)=s(-20)=-s20...

(s80s80s10)/(((-s20+c10)/2)-c80c80c10)=

...s20=2s10c10...

(s80s80s10)/(((-2s10c10+c10)/2)-c80c80c10)=

...no numerador, s80=c10 e simplifica ...

(c10s80s10)/(((-2s10c10+c10)/2)-c80c80c10)=

(s80s10)/((1/2)-s10-c80c80)=

... 1/2 = s30 ...

(s80s10)/(s30-s10-c80c80)=

... s30-s10 = 2c20s10 ...

(s80s10)/(2c20s10-c80c80)=

... c80=s10 e simplifica ...

s80/(2c20-c80)

... e aqui eu parei. Douglas, agora você continua :)


Ricardo Bittencourt   http://www.mundobizarro.tk
[EMAIL PROTECTED]   tenki ga ii kara sanpo shimashou
-- União contra o forward - crie suas proprias piadas --

=
Instruções para entrar na lista, sair da lista e usar a lista em
http://www.mat.puc-rio.br/~nicolau/olimp/obm-l.html

=


=
Instruções para entrar na lista, sair da lista e usar a lista em
http://www.mat.puc-rio.br/~nicolau/olimp/obm-l.html
=


[obm-l] RES: [obm-l] Re: [obm-l] Simplificação Trigonométria

2004-04-04 Por tôpico Douglas Ribeiro Silva
Olá pessoal... estou com um problema para simplificar uma expressão
trigonométrica mas não consegui encontrar relações para chegar na
simplificação. A expressão é:

 

tg²(80°) * [tg(60°) – tg(50°)] / tg²(80°) – [tg(50°) * tg(60°)]

 

Se isso ajuda em alguma coisa, eu sei que a resposta é tg(30°)

 

Agradeço qualquer ajuda na questão.

 

Um abraço, Douglas Ribeiro Silva

-Mensagem original-
De: [EMAIL PROTECTED] [mailto:[EMAIL PROTECTED] Em
nome de [EMAIL PROTECTED]
Enviada em: sexta-feira, 2 de abril de 2004 10:33
Para: [EMAIL PROTECTED]
Assunto: [obm-l] Re: [obm-l] Simplificação Trigonométria

Escrevbe de novo por favor

-- Mensagem original --

Alguém pelo menos tentou fazer a simplificação que eu mandei na
terça-feira? Se tentou por favor me envie um e-mail a parte da lista
pelo menos pra eu ter idéia de quantas pessoas tentaram.
 
Um abraço, Douglas Ribeiro Silva


-O QUE FAREMOS AMANBHA A NOITE CEREBRO?

-AQUILO QUE FAZEMOS TODAS AS NOITES, PINKY:
TENTAR CONQUISTAR O MUNDO!!


--
Use o melhor sistema de busca da Internet
Radar UOL - http://www.radaruol.com.br





=
Instruções para entrar na lista, sair da lista e usar a lista em
http://www.mat.puc-rio.br/~nicolau/olimp/obm-l.html

=


=
Instruções para entrar na lista, sair da lista e usar a lista em
http://www.mat.puc-rio.br/~nicolau/olimp/obm-l.html
=


[obm-l] Conjuntos

2004-04-04 Por tôpico Douglas Ribeiro Silva








Olá pessoal, gostaria de ver a
resolução de vocês para estes dois problemas de conjuntos que me foram
apresentados:



1) Em Porto Alegre foi feita uma pesquisa
com a população sobre suas bebidas prediletas e habituais, e os
resultados foram os seguintes:



60% das pessoas toma
refrigerante;

70% toma vinho;

80% toma café;

90% toma chimarrão



Verificou-se ainda que nenhuma das pessoas consome as quatro bebidas.



Qual a porcentagem de pessoas de Porto Alegre que toma
refrigerante ou vinho?



2) Numa turma do Bacharelado em Matemática 72% dos alunos
gostam de Cálculo, 68% gostam de Teoria dos Números e 60% gostam de Álgebra Linear.
Sabendo que 10% dos alunos não gostam de nenhuma das 3
matérias, qual o máximo percentual de alunos que gostam de duas e somente duas
das matérias acima.



3) 100 pessoas fizeram uma aposta(BOLÃO)
sobre os 3 primeiros lugares do campeonato pernambucano. Os times que estavam
disputando eram o Náutico, o Sport e o Santa Cruz. Em cada aposta, o apostador indica qual será a
classificação dos 3 times.



Das 100 apostas, 48 apontavam o Náutico com vencedor, 51
apontavam o Santa Cruz como 2º colocado, 34
apostadores indicaram o Sport como campeão e 16
apostaram no Sport como segundo colocado.



Sabendo que neste ano o Sport
venceu o campeonato, ficando o náutico em com a segunda posição seguido pelo Santa Cruz em terceiro, Determine qual o prêmio máximo
que um apostador ganhou neste bolão sabendo que cada pessoa entrou com R$1,50.








[obm-l] Simplificação Trigonométria

2004-03-26 Por tôpico Douglas Ribeiro Silva








Alguém pelo menos tentou fazer a simplificação que eu mandei
na terça-feira? Se tentou por favor me envie um e-mail
a parte da lista pelo menos pra eu ter idéia de quantas pessoas tentaram.



Um abraço, Douglas Ribeiro Silva








[obm-l] RES: [obm-l] Simplificação trigonométrica

2004-03-24 Por tôpico Douglas Ribeiro Silva
Acho que vc colocou algo errado na expressão Ricardo, pois pode-se ver
facilmente que essa expressão nunca daria um resultado negativo.
Tg²(80) é 32,algumacoisa tg60 - tg50 da um numero positivo também. Veja
que no denominador isso também é verdade. Não pode ser negativo de jeito
nenhum.

Um abraço, Douglas Ribeiro Silva

-Mensagem original-
De: [EMAIL PROTECTED] [mailto:[EMAIL PROTECTED] Em
nome de Ricardo Bittencourt
Enviada em: quarta-feira, 24 de março de 2004 12:32
Para: [EMAIL PROTECTED]
Assunto: Re: [obm-l] Simplificação trigonométrica

Douglas Ribeiro Silva wrote:

 tg²(80°) * [tg(60°) – tg(50°)] / tg²(80°) – [tg(50°) * tg(60°)]
 Se isso ajuda em alguma coisa, eu sei que a resposta é tg(30°)

Deve ter alguma coisa errada no enunciado,
porque numericamente sua expressão dá -1.523, e
tan(30)=0.577


Ricardo Bittencourt   http://www.mundobizarro.tk
[EMAIL PROTECTED]   tenki ga ii kara sanpo shimashou
-- União contra o forward - crie suas proprias piadas --

=
Instruções para entrar na lista, sair da lista e usar a lista em
http://www.mat.puc-rio.br/~nicolau/olimp/obm-l.html

=


=
Instruções para entrar na lista, sair da lista e usar a lista em
http://www.mat.puc-rio.br/~nicolau/olimp/obm-l.html
=


[obm-l] Simplificação trigonométrica

2004-03-23 Por tôpico Douglas Ribeiro Silva








Olá pessoal... estou com um
problema para simplificar uma expressão trigonométrica mas não consegui
encontrar relações para chegar na simplificação. A expressão é:



tg²(80°) * [tg(60°) 
tg(50°)] / tg²(80°)  [tg(50°) * tg(60°)]



Se isso ajuda em alguma coisa, eu sei que a resposta é tg(30°)



Agradeço qualquer ajuda na questão.



Um abraço, Douglas Ribeiro Silva








RES: [obm-l] Sequencia Geometrica?

2004-03-12 Por tôpico Douglas Ribeiro Silva
Perceba que...

A(1) = 0 + 1
A(2) = 6 + 1
A(3) = 18 + 1
A(4) = 36 + 1

Essa seqüência é uma PA de segunda ordem, já que A2 - A1, A3 - A2 e A4 -
A3 formam nessa ordem uma PA de primeira ordem.

Pode-se dizer então que há um polinômio an² + bn + c que define a
seqüência.
Descobre-se então este polinômio a partir do que se conhece da
seqüência:

a + b + c = 1
4a + 2b + c = 7
9a + 3b + c = 19

Resolvendo o sistema, temos que a = 3, b = -3 e c =1
Logo:

An = 3n² -3n + 1

Um abraço, Douglas Ribeiro Silva

-Mensagem original-
De: [EMAIL PROTECTED] [mailto:[EMAIL PROTECTED] Em
nome de Qwert Smith
Enviada em: sexta-feira, 12 de março de 2004 11:44
Para: [EMAIL PROTECTED]
Assunto: [obm-l] Sequencia Geometrica?

Aqui vai um problema que acho que pode ser descrito como uma sequencia.

Achar o numero maximo de areas formadas pela intercecao de n triangulos 
assim temos
A(1) = 1 ( 1 triagulo, uma area )
A(2) = 7 ( 2 triangulos, 7 areas como a estrela de david )
A(3) = 19 ( eu contei 19, mas vale a pena conferir )
...
A(n) = ?

O problema original era quantas areas sao formadas por (1 + 10^(um
numero 
ridicularmente grande))

Alguma dica?

_
Create a Job Alert on MSN Careers and enter for a chance to win $1000! 
http://msn.careerbuilder.com/promo/kaday.htm?siteid=CBMSN_1Ksc_extcmp=J
S_JASweep_MSNHotm2


=
Instruções para entrar na lista, sair da lista e usar a lista em
http://www.mat.puc-rio.br/~nicolau/olimp/obm-l.html

=


=
Instruções para entrar na lista, sair da lista e usar a lista em
http://www.mat.puc-rio.br/~nicolau/olimp/obm-l.html
=


[obm-l] Combinatoria Classica

2004-03-02 Por tôpico Douglas Ribeiro Silva








Olá pessoal, eu estava analisando um caso desses de combinatória
de caminhos, do tipo: considerando-se uma Matriz
4x4, pede-se o numero de probabilidades de chegar do A(41) ao A(14). Nesse caso
a resposta da 20. Me interessei em saber disso quando dando uma olhada no
arquivo da lista encontrei este problema:

http://www.mat.puc-rio.br/~nicolau/olimp/obm-l.200312/msg00137.html

Como mostra a mensagem, existe um conceito de triangulo de
pascal aí no problema, onde o numero de caminhos para se chegar numa
determinada casa é dado pela soma da quantidade de caminhos das casas à
esquerda e abaixo delas. Gostaria de saber como poder provar isso generalizando
pra uma casa qualquer da matriz.



Já agradeço a atenção. []s, Douglas








RES: [obm-l] geometria

2004-02-25 Por tôpico Douglas Ribeiro Silva
Mas como seria feita a medida desses angulos Nicolau? Já que num
triangulo esférico a soma dos ângulos é sempre maior que 180? Pq se
fossem os ângulos do plano relativo aos 3 pontos que formam o triangulo
seria mais fácil, especialmente no caso do tetraedro, onde A = B = C =
60, mas no caso da esfera eu pelo menos não faço idéia de como se faz.

Aproveitando o problema... Gostaria de saber se há como a generalização
dele: Dado um triedro com vértice no centro de uma esfera de raio R,
determinar o seu volume em função dos 3 ângulos formados entre as
semi-retas que formam o triedro. Acho que seria bem interessante,
cheguei a elaborar algumas idéia sobre isso, mas não tive grandes
êxitos.

Um abraço, Douglas Ribeiro Silva

-Mensagem original-
De: [EMAIL PROTECTED] [mailto:[EMAIL PROTECTED] Em
nome de Nicolau C. Saldanha
Enviada em: quarta-feira, 25 de fevereiro de 2004 19:50
Para: [EMAIL PROTECTED]
Assunto: Re: [obm-l] geometria

Um fato que ajuda muito é o seguinte. Um triângulo esférico é um pedaço
da esfera de raio 1 limitaedo por três segmentos que são pedaços de
círculos máximos. Um triângulo esférico tem três ângulos A, B, C.
A área deste triângulo é A + B + C - Pi (onde A, B, C são medidos
em radianos).

On Wed, Feb 25, 2004 at 08:42:54PM +, [EMAIL PROTECTED] wrote:
 DESAFIO!!
 @4 esferas iguais de raio r estão se tangenciando de forma que a
ligação de
 seus centros formem um tetraedro. O tetraedro “corta” um certo volume
de
 cada esfera, qual é o valor desse volume em função de r?

Tome r = 1. Os ângulos entre faces de um tetraedro regular são iguais
a A = 2 arc sen(sqrt(3)/3) ~= 1.230959418. Então a área do triângulo
esférico
contido no tetraedro é SA = 3*A - Pi ~= 0.551285599. O volume é 1/3
disso
(pois o volume da esfera de raio 1 é 1/3 da sua área) logo
A - Pi/3 ~= 0.1837618663.

Se o raio tiver outro valor é só multiplicar por r^3.

Observe que isto é um pouco menos de 1/20 do volume da esfera
(que dá 4*Pi/(3*20) ~= 0.2094395103.

 @5 esferas iguais de raio r estão se tangenciando da forma que a
ligação de
 seus centros forme uma pirâmide de base quadrática com todas as
arestas
 iguais. Haverá 2 tipos de volumes cortados pelas esferas: o volume que
as 4
 esferas da base quadrática “corta” da pirâmide e o volume que a esfera
do
 topo “corta” da mesma. Qual é o valor desses dois volumes em função de
r?

Aqui os centros das suas 5 esferas são 5 dos 6 vértices de um octaedro
regular então o volume do topo é o dobro de cada um dos volumes da base.
Da mesma forma o triângulo esférico que aparece na base tem ângulos 
B, B e 2B, onde B é o ângulo entre uma face do octaedro e o plano
que passa por 4 dos seus vértices. Mas B é igual ao ângulo formado 
pelos vetores (1,1,1) e (0,0,1) (que são perpendiculares a uma face e a
um plano se os vértices do octaedro forem (+-sqrt(2),0,0),
(0,+-sqrt(2),0), (0,0,+-sqrt(2)) para que a aresta seja 2)
logo B = arc cos(sqrt(3)/3) ~= 0.9553166180. Também dava para ver que
A/2 + B = Pi/2 olhando como octaedros e tetraedros se encaixam para
encher o espaço (tome todos os pontos de coordenadas inteiras com soma
par e ligue pontos a uma distância sqrt(2)). Mas o fato é que a área
do nosso triângulo esférico é 4*B - Pi ~= 0.679673818 e o volume é
(4*B - Pi)/3 ~= 0.2265579393. A área no topo é o dobro, como já
dissemos,
SB = 8*B - 2*Pi ~= 1.359347636 e o volume é (8*B - 2*Pi)/3 ~=
0.4531158786.

Observe que 6*SB + 8*SA = 4*Pi, coerentemente com aquela maneira de
encher
o espaço com octaedros e tetraedros: há 6 octaedros e 8 tetraedros ao
redor
de cada vértice.

 @Se do volume da pirâmide quadrática acima for “cortado” todos os
volumes
 formado pelas 5 esferas, parte somente de dentro da pirâmide, sobrará
um
 volume central não “cortado”.

O volume da pirâmide (meio octaedro) é claramente 4*sqrt(2)/3 ~=
1.885618082.
Este volume central é portanto 4*sqrt(2)/3 - 8*B + 2*Pi ~= 0.526270446.

 Se o volume central fosse
 necessariamente “distribuído” para as 5 esferas, como seria feito a
 distribuição? Ela seria proporcional à área superficial da parte
esférica de
 dentro da pirâmide ou ao volume que cada esfera “corta” da pirâmide?

Esta parte eu não entendi. Minha única observação é que os volumes e
áreas
são trivialmente proporcionais, como já vimos.

[]s, N.

=
Instruções para entrar na lista, sair da lista e usar a lista em
http://www.mat.puc-rio.br/~nicolau/olimp/obm-l.html

=


=
Instruções para entrar na lista, sair da lista e usar a lista em
http://www.mat.puc-rio.br/~nicolau/olimp/obm-l.html
=


RES: [obm-l] Descubra os lados do Triangulo

2004-02-22 Por tôpico Douglas Ribeiro Silva









Olá Rafael. Já que você tocou no assunto e
mostrou domínio sobre ele... Poderia esclarecer-me por favor
do que se trata o Método de Ferrari? Me interessei em
saber sobre isso.



Um abraço, Douglas Ribeiro Silva



-Mensagem original-
De: [EMAIL PROTECTED]puc-rio.br
[mailto:[EMAIL PROTECTED] Em nome
de Rafael
Enviada em: sábado, 21 de
fevereiro de 2004 17:29
Para: [EMAIL PROTECTED]puc-rio.br
Assunto: Re: [obm-l] Descubra os
lados do Triangulo 





Claro que sim, Pérsio. Cláudio,
aliás, enviou uma para lista já. No entanto, não creio que ela seja exatamente
o que se possa chamar de trivial, pois envolve alguns artifícios
geométricos e a manipulação algébrica de várias expressões. Evidentemente, ainda
é mais acessível do que o Método de Ferrari, que torna o problema
puramente algébrico, pois o único conceito de Geometria que se usa é a
correspondência dos ângulos,amparada
pelaTrigonometriatambém.











De qualquer forma, com mais ou menos
trabalho braçal, chegou-sea mesma solução...











Abraços,











Rafael de A. Sampaio



















- Original Message - 





From: persio ca 





To: [EMAIL PROTECTED]






Sent: Saturday,
February 21, 2004 10:55 AM





Subject: Re: [obm-l]
Descubra os lados do Triangulo 











Será que existe uma
solução geometrica trivial para este problema ?











Pérsio












RES: Spam Alert: [obm-l] Por Favor....

2004-02-22 Por tôpico Douglas Ribeiro Silva








Só complementando o e-mail do Pacini, eu aprendi isso aí como PAG(Progressão
Aritmética e Geometria), que é uma progressão na qual os numeradores formam uma
PA, e os denominadores uma PG. Não sei em qual livro meu professor tomou como
base para dar o assunto, mas certamente deve ter em livros que tratam de
progressão.



ACHO que pra toda PAG é válida a
propriedade dita pelo Pacini de Convergência. Não tenho
certeza disso. Se alguém da lista puder enviar uma prova eu ficaria bastante
grato.



-Mensagem original-
De: [EMAIL PROTECTED]puc-rio.br
[mailto:[EMAIL PROTECTED] Em nome
de Pacini bores
Enviada em: domingo, 22 de
fevereiro de 2004 10:59
Para: [EMAIL PROTECTED]puc-rio.br; [EMAIL PROTECTED]puc-rio.br
Assunto: Re: Spam Alert: [obm-l]
Por Favor



At 00:35 22/2/2004, [EMAIL PROTECTED] wrote:



Alguém poderia me ajudar nesse
problema de P.G infinita. Não consigo achar o diabo da razão:

1+2/2+3/4+4/8+5/16.

Grato
Junior 



O ideal neste tipo de questão é dividir
toda a expressão pela razão da PG do denominador :





S = 1+2/2+3/4+4/8+5/16. (1)


S/2 = 1/2 + 2/4 + 3/8 + 4/16 + 5/32 +... (2)


Faz (1) -(2) : S/2 = 1 + 1/2 + 1/4 + 1/8 + ...

 S/2 = 2 , donde S = 4 .

[]´s Pacini


Nota : 1)é importante observar que há necessidade
de mostrar que estas séries são convergentes
2)Na solução do Luiz França houve pequeno
engano na conta final 

2 +1 + 1/2 +... = 4 e não 3 , ok ?








RES: [obm-l] restos

2004-02-22 Por tôpico Douglas Ribeiro Silva








No houve erro no
enunciado do 2 problema?

Como um numero
pode ser dividido por 3 dar resto 3?!



Se fosse qualquer outro numero, eu diria
que a resposta era o MMC desses 3 numeros
+ o resto



Um abrao, Douglas Ribeiro Silva



-Mensagem original-
De: [EMAIL PROTECTED]puc-rio.br
[mailto:[EMAIL PROTECTED] Em nome
de [EMAIL PROTECTED]com
Enviada em: domingo, 22 de
fevereiro de 2004 14:57
Para: [EMAIL PROTECTED]puc-rio.br
Assunto: [obm-l] restos



Ola pessoal, 


Como resolver estes ? 


1.Qual o maior numero inteiro quepodemos somar ao
dividendo 
de um divisao, onde o divisor eh 13 e o resto eh 2, sem 
que o quociente sofra alteracao? 



2.Qual o menor numero que dividido por 3, 11, e 51
deixa 
sempre resto 3? 

ps: Nao conheco o TCR (teorema chines do resto)








RES: RES: [obm-l] torres

2004-02-20 Por tôpico Douglas Ribeiro Silva








Concordo
inteiramente com o seu ponto de vista. Só não entendi pq vc
mandou esse e-mail em resposta ao meu =P



Um abraço, Douglas Ribeiro Silva



-Mensagem original-
De: [EMAIL PROTECTED]puc-rio.br
[mailto:[EMAIL PROTECTED] Em nome
de [EMAIL PROTECTED]com
Enviada em: sexta-feira, 20 de
fevereiro de 2004 04:03
Para: [EMAIL PROTECTED]puc-rio.br
Assunto: Re: RES: [obm-l] torres



Ola, 

UMA COISA eh voce pedir para alguem colocar 8 rainhas
em um trabuleiro nao havendo ataque uma nas outras (MATEMATICA RECREATICA)
OUTRA COISA eh calcular de quantos modos isso eh possivel (PROBABILIDADE). 
Sao areas diferentes na matematica, mas cada uma tem seus adeptos e amantes. 
Da mesma forma que dizer para alguem montar o cubo Rubick (MATEMATICA
RECREATIVA (UTILIZANDO APENAS RACIOCINIO) E/OU TEORIA DOS GRUPOS) ou calcular o
numero total de diferentes configuracoes (PROBABILIDADE). Ha uma diferenca
estetica ao meu ver. 



Em uma mensagem de 20/2/2004 03:53:37 Hora padrão
leste da Am. Sul, [EMAIL PROTECTED]br escreveu: 






Na verdade o problema que ele passou é o
mesmo problema das rainhas. De quantas formas podemos colocar essas rainhas no
tabuleiro? 

-Mensagem original- 
De: [EMAIL PROTECTED]puc-rio.br
[mailto:[EMAIL PROTECTED] Em nome
de [EMAIL PROTECTED]com 
Enviada em: sexta-feira, 20 de
fevereiro de 2004 02:22 
Para: [EMAIL PROTECTED]puc-rio.br 
Assunto: Re: [obm-l] torres 

Ola, 

Tem certeza que digitou corretamente o enunciado ? 

Seria: 

De forma que 2 torres nao estejam na mesma linha 

OU 

De forma que as 8 torres nao se ataquem ? 


Ps: Eu ja vi um bem interessante: Coloque 8 rainhas em
um tabuleiro sem que nenhuma ataque as outras  




Em uma mensagem de 19/2/2004 17:33:00 Hora padrão
leste da Am. Sul, [EMAIL PROTECTED]br escreveu: 





De quantas maneiras podemos arrumar 8 torres iguais em um tabuleiro de forma 
que duas torres não estejam na mesma linha, coluna ou diagonal? 
















RES: [obm-l] Descubra os lados do Triangulo

2004-02-19 Por tôpico Douglas Ribeiro Silva








Ah... ainda
bem que alguem mandou um e-mail sobre esse problema
porque eu já estava me esquecendo de perguntar isso...



Vi a bela resolução do Cláudio para este
problema, mas heis a questão... e
se em vez de um dos lados do quadrado estar sobreposto à hipotenusa, tivéssemos
dois lados do quadrado sobrepostos aos catetos?



Foi desse modo que eu pensei inicialmente
e tentei resolver a questão, mas sempre caí numa eq. de grau 3 ou 4. Pensei que ia cair numa biquadrada
bonitinha, mas os termos não se anularam. Gostaria que alguém mandasse a
resolução desse jeito, se é que é possível resolver sem usar Cardano-Tartaglia.



Um abraço, Douglas Ribeiro



-Mensagem original-
De: [EMAIL PROTECTED]puc-rio.br
[mailto:[EMAIL PROTECTED] Em nome
de [EMAIL PROTECTED]com
Enviada em: quinta-feira, 19 de
fevereiro de 2004 00:58
Para: [EMAIL PROTECTED]puc-rio.br
Assunto: Re: [obm-l] Descubra os
lados do Triangulo 



Essa questao ja foi resolvida pessoal ! 
Guilherme, um lado do quadrado sobrepoe-se a hipotenusa. 



Em uma mensagem de 18/2/2004 23:56:59 Hora padrão
leste da Am. Sul, [EMAIL PROTECTED]br escreveu: 






Como um quadrado increve um triangulo?... 
Lados sobrepostos?... 
Um vértice do quadrado tocando um lado do triângulo? 

persio ca [EMAIL PROTECTED]com.br wrote: 




Pessoal 

Alguem consegue resolver este problema sem usar cardano tartaglia, somente
usando pura geometria. 

Considere um triangulo retangulo com hipotenusa 12 e com um quadrado
inscrito de lado 4. A pergunta qual é o valor total de
seus catetos ? 

Persio 














RES: [obm-l] torres

2004-02-19 Por tôpico Douglas Ribeiro Silva








Na verdade o problema que
ele passou é o mesmo problema das rainhas. De quantas formas podemos colocar
essas rainhas no tabuleiro?



-Mensagem original-
De: [EMAIL PROTECTED]puc-rio.br
[mailto:[EMAIL PROTECTED] Em nome
de [EMAIL PROTECTED]com
Enviada em: sexta-feira, 20 de
fevereiro de 2004 02:22
Para: [EMAIL PROTECTED]puc-rio.br
Assunto: Re: [obm-l] torres



Ola, 

Tem certeza que digitou corretamente o enunciado ? 

Seria: 

De forma que 2 torres nao estejam na mesma linha 

OU 

De forma que as 8 torres nao se ataquem ? 


Ps: Eu ja vi um bem interessante: Coloque 8 rainhas em
um tabuleiro sem que nenhuma ataque as outras  




Em uma mensagem de 19/2/2004 17:33:00 Hora padrão
leste da Am. Sul, [EMAIL PROTECTED]br escreveu: 






De quantas maneiras podemos arrumar 8 torres iguais em
um tabuleiro de forma 
que duas torres não estejam na mesma linha, coluna ou diagonal? 










[obm-l] Geometria 1 e 2

2004-02-15 Por tôpico Douglas Ribeiro Silva








Olá pessoal.



Vendo as mensagens anteriores, eu me interessei em adquirir
os livros Geometria 1 e 2 do prof. Morgado
e do prof. Eduardo Wagner, mas procurei um pouco Internet e não encontrei sites que vendessem ele. Se alguém puder me indicar algum site em que eu possa compra-los
eu ficaria muito grato. Alias, gostaria de saber também se é possível adquiri-los
aqui na UFPE, pois eu vi no site da SMB que tem
professores aqui que vendem livros de lá.



Um abraço, Douglas








[obm-l] RES: [obm-l] Questão de conjuntos

2004-02-13 Por tôpico Douglas Ribeiro Silva
O total de pessoas que le revistas é:

Pessoas que lêem SOMENTE 1 revista +
Pessoas que lêem EXATAMENTE 2 revistas +
Pessoas que lêem AS TRES revistas

Tudo isso somado tem que dar 81

Logo: 61 + 17 + X = 81

X = 3



-Mensagem original-
De: [EMAIL PROTECTED] [mailto:[EMAIL PROTECTED] Em
nome de thor-oliveira
Enviada em: sexta-feira, 13 de fevereiro de 2004 18:31
Para: obm-l
Assunto: [obm-l] Questão de conjuntos


  Como se faz essa?


Numa pesquisa para se avaliar a leitura de 
tres revistas A , B e C , descobriu-se que 81 pessoas 
leem , pelo menos , uma das revistas; 61 pessoas leem 
somente uma delas e 17 pessoas leem duas das tres 
revistas.Assim sendo , o numero de pessoas mais 
informadas dentre as 81 eh : 

a) 3
b)5
c)12
d)29
e)37
 

__
Acabe com aquelas janelinhas que pulam na sua tela.
AntiPop-up UOL - É grátis!
http://antipopup.uol.com.br/




=
Instruções para entrar na lista, sair da lista e usar a lista em
http://www.mat.puc-rio.br/~nicolau/olimp/obm-l.html

=


=
Instruções para entrar na lista, sair da lista e usar a lista em
http://www.mat.puc-rio.br/~nicolau/olimp/obm-l.html
=


RES: [obm-l] P.G.

2004-02-12 Por tôpico Douglas Ribeiro Silva








Pn = (a1^n)*q^(n(n-1)/2)
Então:

2^39 = (1^n)*2^[(1/2)*n(n-1)/2]

2^39 = 2^[n(n-1)/4]

Logo: 39
= n(n-1)/4

n² -n + 156 = 0

n = 1
+-(25)/2

n=13

Acho que
é isso...

Abraços,
Douglas Ribeiro



-Mensagem original-
De: [EMAIL PROTECTED]puc-rio.br
[mailto:[EMAIL PROTECTED] Em nome
de pedro rajão
Enviada em: quinta-feira, 12 de
fevereiro de 2004 23:24
Para: [EMAIL PROTECTED]puc-rio.br
Assunto: [obm-l] P.G.







Uma
progressão geométrica tem 1° termo igual a 1 e r=2^1/2
. Se o produto dos termos é 2^39 .: o
nº de termos é = a ?














MSN Hotmail, o maior
webmail do Brasil. Faça o seu
agora. 






=
Instruções para entrar na lista, sair da lista e usar a lista em
http://www.mat.puc-rio.br/~nicolau/olimp/obm-l.html
=

[obm-l] RES: [obm-l] O Perímetro do Triângulo

2004-02-12 Por tôpico Douglas Ribeiro Silva









Olá Renato!



O problema da pra sair usando o conceito
de base média dos triângulos...



Tome o triangulo ADC.

O segmento PR é base média desse
triangulo, considerando-se AD como base, já que R é ponto médio de DC e P ponto
médio de AC. Sendo assim, PR vale 3m.

Analogamente, QR é base média de DBC.
Valendo também 3m.



Agora a importância da soma dos ângulos valerem
120°!



PR e QR são paralelos a AD e BC. Então o ângulo
PRQ pode ser medido com a soma dos ângulos internos desse triangulo: ^P + ^R + ^Q
= 180.

Mas ^P + ^Q = 120°. Então ^R = 60°

Se ficar difícil visualizar que ^P + ^Q = ^A + ^B, trace uma paralela a AB passando por
P ou Q, fica mais fácil de ver isso.

Nesse caso temos em PRQ um
triangulo isosceles, com os
lados iguais medindo 3, e o ângulo entre esses lados medindo 60°.

Percebe-se então que este
triangulo na verdade é eqüilátero, o que faz o segmento PQ valer 3m também!



O perímetro vale 9.



Quaisquer problemas nas passagens é só
avisar.



Um abraço, Douglas Ribeiro Silva



-Mensagem original-
De: [EMAIL PROTECTED]puc-rio.br
[mailto:[EMAIL PROTECTED] Em nome
de Renato de Brito
Enviada em: quinta-feira, 12 de
fevereiro de 2004 01:20
Para: [EMAIL PROTECTED]puc-rio.br
Assunto: [obm-l] O Perímetro do
Triângulo





Gostaria da ajuda dos amigos nesta
questão.











ABCD é um quadrilátero onde AD=BC e
os ângulos DAB e ABC somam 120º.Calcule o perímetro do triângulo PQR, sabendo
que P é o ponto médio da diagonal AC, Q é o ponto médio da diagonal BD, R é o
ponto médio do lado CDe que AD=6m. 










RES: RES: RES: [obm-l] area de triangulo

2004-02-11 Por tôpico Douglas Ribeiro Silva
Guilherme, creio que quando vc perguntou pq as circunferências são
tangentes é pelo mesmo motivo que Fabio perguntou isso...

Fabio, a rigor era pra ele ter dito no enunciado que elas são tangentes
entre si mesmo. No entanto quando fui resolver o problema supus isso,
porque do contrário o problema iria ter infinitas respostas. Pode-se
colocar infinitas circunferências de raio 4 dentro de um triangulo
arbitrário, mas se elas não forem tangentes entre si o problema teria
infinitas respostas. O lado do triangulo no caso delas serem tangentes
entre si é aprox. 21,86cm. Usando-se lados maiores que isso as
circunferências teriam espaços de distancias indefinidas entre si(que
não foram citados no enunciado), o que deixaria com infinitas respostas.
Usando lados que fossem menores que isso elas estariam se interceptando
em pontos que também não teriam sido citados no enunciado.

É isso... =)

Um abraço, Douglas Ribeiro


-Mensagem original-
De: [EMAIL PROTECTED] [mailto:[EMAIL PROTECTED] Em
nome de Fabio Henrique
Enviada em: quarta-feira, 11 de fevereiro de 2004 18:01
Para: [EMAIL PROTECTED]
Assunto: Re: RES: RES: [obm-l] area de triangulo

Não faltou dizer que as circunferências são tangentes entre si duas a
duas? 

Em 10 Feb 2004, [EMAIL PROTECTED] escreveu: 

![endif]-- 
 
 -- 
 
![endif]-- 
 
 Acho que é isso: 
 
 http ://www. klystron . kit . net /triangulo.jpg 
 
 -Mensagem original- 
 
 De: [EMAIL PROTECTED] puc -rio. br 
[mailto:[EMAIL PROTECTED] Em nome 
de Guilherme Carlos Moreira e Silva 
 
 Enviada em: terça-feira, 10 de 
fevereiro de 2004 19:52 
 
 Para: [EMAIL PROTECTED] puc -rio. br 
 
 Assunto: Re: RES: [obm-l] area de 
triangulo 
 
 bem 
ñ entendi bem o enunciado da questao e por isto ela me pareceu facil 
 
 poderia 
mandar uma figura? 
 
 Douglas Ribeiro Silva 
 com. br wrote: 
 
 Salvo engano sua área é 32[2sqrt(3) + 3] 
 
 Bom, o 
ângulo formado entre um lado do triangulo e um dos vértices do
triangulo 
até o 
centro da circunferência mais próxima desse vértice é 30°. Desse centro
até 
o 
lado são 4cm, pois ela é tangente. Como o ângulo é de 30° então do
ponto de 
tangência até o vértice do triangulo vai ser 4sqrt(3) cm. Isso
obviamente 
vale 
pro outro lado do triângulo. Logo pra descobrir o tamanho do lado falta
só 
o 
meio do lado que é um segmento de 8cm, formado pela união dos 
centros das circunferências internas de raio 4cm. Logo o lado do
triângulo 
vale 
4sqrt(3) + 8 + 4sqrt(3) = 8(sqrt (3 ) +1) cm. 
 
 Daí: 
 
 A= 
L²sqrt (3 )/4 
 
 Desenvolvendo 
dá 32[2sqrt(3) + 3] cm² 
 
 Avisem-me 
se por acaso saiu algo errado... Douglas Ribeiro 
 
 -Mensagem original- 
 
 De: [EMAIL PROTECTED] puc -rio. br [mailto:[EMAIL PROTECTED] 
 Em nome de [EMAIL PROTECTED] com 
 
 Enviada em: terça-feira, 10 de 
fevereiro de 2004 00:29 
 
 Para: [EMAIL PROTECTED] puc -rio. br 
 
 Assunto: [obm-l] area de triangulo 
 
 Ola pessoal, 
 
Imaginem um triangulo equilatero com 3 circunferencias 
de raio 4 cm inscritas neste triangulo. Cada lado do triangulo eh
tangente 
a 2 circunferencia . Qual a area do triangulo 
? 
 
 Yahoo! 
Mail - 6MB, anti-spam e antivírus gratuito. Crie 
sua conta agora! 
 
-- 

_
Voce quer um iGMail protegido contra vírus e spams? 
Clique aqui: http://www.igmailseguro.ig.com.br



=
Instruções para entrar na lista, sair da lista e usar a lista em
http://www.mat.puc-rio.br/~nicolau/olimp/obm-l.html
=


RES: [obm-l] O PARADOXO DE BERTRAND!

2004-02-11 Por tôpico Douglas Ribeiro Silva
Não tenho certeza sobre o tal paradoxo, mas ao que me parece seria
relativo ao processo de obtenção dessa corda.

Escolhendo ao acaso uma corda de uma circunferência, qual é a
probabilidade de que ela seja maior que o lado do triângulo equilátero
inscrito nessa 
circunferência?

Se escolhermos uma corda já pronta dentro da circunferência a
probabilidade é diferente de considerar essa corda(que não deixa de ser
um segmento de reta) formada pela união de 2 pontos escolhidos
aleatoriamente na circunferência.

No caso da corda já pronta acho que a probabilidade é 1/2. Traça-se o
diâmetro da circunferência e considerando-se a corda perpendicular a
este diâmetro, ela teria somente metade do segmento para ser maior que o
lado do triangulo eqüilátero inscrito.

Agora, se formarmos a corda por 2 pontos escolhidos aletoriamente
teremos 2 possibilidades:

a) o primeiro ponto cai dentro da circunferência concêntrica à citada no
enunciado, e com raio de metade da mesma(Não sei se usei o português
corretamente... O primeiro ponto cai dentro de uma circunferência de
raio 2, se a circunferência citada no enunciado for de raio 4. Estas
duas são concentricas). Se assim for, então o próximo ponto poderá cair
onde quiser que ele será maior que o lado do triangulo eqüilátero
inscrito.

b) primeiro ponto cai dentro da circunferência concêntrica à citada no
enunciado, e com raio de metade da mesma. Então teria-se que calcular a
área em que o outro ponto poderia ser posto para satisfazer a condição
do enunciado. Na verdade esse cálculo acho que é bem complicado... pois
pelo que deu pra perceber existe uma probabilidade mínima e uma
probabilidade máxima, que seriam respectivamente se o primeiro ponto
caísse em cima da circunferência de raio maior, ou se o primeiro ponto
caísse em cima da de raio menor. Por sinal eu gostaria de saber de
alguém como seria o calculo dessa probabilidade média. Seria tomando uma
área média? Ou fazendo a media das probabilidades mínima e máxima? Ou
tanto faz? Heheheh eu realmente fiquei na duvida

Não sei se é isso que realmente define o tal paradoxo, mas pelo que deu
a entender é isso. Bem interessante!

Abraços, Douglas Ribeiro

-Mensagem original-
De: [EMAIL PROTECTED] [mailto:[EMAIL PROTECTED] Em
nome de Nicolau C. Saldanha
Enviada em: quarta-feira, 11 de fevereiro de 2004 11:39
Para: [EMAIL PROTECTED]
Assunto: Re: [obm-l] O PARADOXO DE BERTRAND!

On Tue, Feb 10, 2004 at 08:26:56PM -0300, [EMAIL PROTECTED] wrote:
 Turma! Alguma idéia a respeito do problema dos dados? Eu,
particularmente,
 continuo na mesma, apesar de achar o raciocínio muito parecido com o
da
 Penélope x Olívia, elucidado recentemente pelo Ralph. Enquanto isso,
vejam
 abaixo um famoso paradoxo em que incrivelmente um problema sobre
 probabilidades passa a ter diversas respostas.

Você quer dizer aquele que eu propus e repito abaixo?
É muito difícil, e a dificuldade é combinatória,
nada a ver com estes problemas de probabilidade com
um raciocínio certo e outro errado.

[]s, N.

PS: O que é o paradoxo de Bertrand?




Tome uma coleção finita de dados. Os dados não precisam ter 6 faces,
o número de faces é um inteiro positivo qq n, e as faces são numeradas
de 1 a n. O valor de n (o número de faces) pode inclusive variar de um
dado
para outro, isto é, estamos misturando dados de vários tipos.
A única restrição é que cada dado deve ser honesto, i.e.,
que em um dado com n faces cada face tem probabilidade 1/n.
Os dados também são independentes uns dos outros, claro.
Vamos jogar todos os dados da coleção e somar todos os números
sorteados:
chamemos esta soma de N.

É bem fácil calcular os valores mínimo e máximo possível de N:
Nmin é o número de dados e Nmax é o número total de faces de todos os
dados.
Seja Nm = (Nmin + Nmax)/2.

Sejam N1  N2 = Nm. Prove que prob(N=N1) = prob(N=N2).


=
Instruções para entrar na lista, sair da lista e usar a lista em
http://www.mat.puc-rio.br/~nicolau/olimp/obm-l.html

=


=
Instruções para entrar na lista, sair da lista e usar a lista em
http://www.mat.puc-rio.br/~nicolau/olimp/obm-l.html
=


RES: [obm-l] area de triangulo

2004-02-10 Por tôpico Douglas Ribeiro Silva








Salvo engano sua área é 32[2sqrt(3)
+ 3]



Bom, o ângulo formado entre um lado do
triangulo e um dos vértices do triangulo até o centro da circunferência mais próxima
desse vértice é 30°. Desse centro até o lado são 4cm, pois ela é tangente. Como
o ângulo é de 30° então do ponto de tangência até o vértice do triangulo vai
ser 4sqrt(3) cm. Isso obviamente vale pro outro lado do triângulo. Logo pra
descobrir o tamanho do lado falta só o meio do lado que é um
segmento de 8cm, formado pela união dos centros das circunferências internas de
raio 4cm. Logo o lado do triângulo vale 4sqrt(3) + 8 + 4sqrt(3) = 8(sqrt(3) +1) cm.



Daí:



A= L²sqrt(3)/4



Desenvolvendo dá 32[2sqrt(3) + 3] cm²



Avisem-me se por acaso saiu algo errado...
Douglas Ribeiro





-Mensagem original-
De: [EMAIL PROTECTED]puc-rio.br
[mailto:[EMAIL PROTECTED] Em nome
de [EMAIL PROTECTED]com
Enviada em: terça-feira, 10 de
fevereiro de 2004 00:29
Para: [EMAIL PROTECTED]puc-rio.br
Assunto: [obm-l] area de triangulo



Ola pessoal, 

Imaginem um triangulo equilatero com 3 circunferencias
de raio 4 cm inscritas neste triangulo. Cada lado do triangulo eh tangente a 2 circunferencia . Qual a area do triangulo
?








RES: RES: [obm-l] area de triangulo

2004-02-10 Por tôpico Douglas Ribeiro Silva








Acho que é isso:

http://www.klystron.kit.net/triangulo.jpg



-Mensagem original-
De: [EMAIL PROTECTED]puc-rio.br
[mailto:[EMAIL PROTECTED] Em nome
de Guilherme Carlos Moreira e Silva
Enviada em: terça-feira, 10 de
fevereiro de 2004 19:52
Para: [EMAIL PROTECTED]puc-rio.br
Assunto: Re: RES: [obm-l] area de
triangulo





bem
ñ entendi bem o enunciado da questao e por isto ela me pareceu facil











poderia
mandar uma figura?

Douglas Ribeiro Silva
[EMAIL PROTECTED]com.br wrote:





Salvo engano sua área é 32[2sqrt(3) + 3]



Bom, o
ângulo formado entre um lado do triangulo e um dos vértices do triangulo até o
centro da circunferência mais próxima desse vértice é 30°. Desse centro até o
lado são 4cm, pois ela é tangente. Como o ângulo é de 30° então do ponto de
tangência até o vértice do triangulo vai ser 4sqrt(3) cm. Isso obviamente vale
pro outro lado do triângulo. Logo pra descobrir o tamanho do lado falta só o
meio do lado que é um segmento de 8cm, formado pela união dos
centros das circunferências internas de raio 4cm. Logo o lado do triângulo vale
4sqrt(3) + 8 + 4sqrt(3) = 8(sqrt(3) +1) cm.



Daí:



A=
L²sqrt(3)/4



Desenvolvendo
dá 32[2sqrt(3) + 3] cm²



Avisem-me
se por acaso saiu algo errado... Douglas Ribeiro





-Mensagem original-
De: [EMAIL PROTECTED]puc-rio.br [mailto:[EMAIL PROTECTED]
Em nome de [EMAIL PROTECTED]com
Enviada em: terça-feira, 10 de
fevereiro de 2004 00:29
Para: [EMAIL PROTECTED]puc-rio.br
Assunto: [obm-l] area de triangulo



Ola pessoal, 

Imaginem um triangulo equilatero com 3 circunferencias
de raio 4 cm inscritas neste triangulo. Cada lado do triangulo eh tangente a 2 circunferencia . Qual a area do triangulo
?











Yahoo!
Mail - 6MB, anti-spam e antivírus gratuito. Crie
sua conta agora!








RES: [obm-l] Cubo de Rubik

2004-02-08 Por tôpico Douglas Ribeiro Silva
Isso realmente é um fato que eu tenho percebido acontecer quase que
rotineiramente aqui na lista. Infelizmente não possuo conhecimentos
matemáticos muito além daqueles de um bom estudante de 2º grau e aluno
de engenharia que acabou de pagar Cálculo 1, por isso não me manifesto
em varias das mensagens. E quando surge uma que eu poderia dar alguma
contribuição, alguém geralmente já o fez antes de mim...

Bom, mas ainda assim gostaria que alguém que tivesse um conhecimento
melhor de combinatória pudesse me mostrar qual o raciocínio usado para o
calculo do cubo de rubik.

Fael, eu vi a página do Cubo4D, já a tinha visto antes e peguei o
programa... é bastante interessante e de certo modo complexo para fazer
o exato movimento que você pensa em fazer hehehehe. Quanto ao Cubo5D não
sei se isso pode ajudar, mas nessa pagina
http://mathworld.wolfram.com/Hypercube.html tem uma imagem do Cubo5D ao
topo  e mostra também as planificação dos Cubos em N dimensões... o
processo de obtenção da planificação é deveras interessante, pois podem
ser obtidas fazendo sucessões de polígonos com lados pares, e utilizando
sucessivas vezes 3 lados consecutivos como arestas de cubos3D situadas
em eixos perpendiculares entre si.


-Mensagem original-
De: [EMAIL PROTECTED] [mailto:[EMAIL PROTECTED] Em
nome de Carlos Maçaranduba
Enviada em: domingo, 8 de fevereiro de 2004 13:46
Para: [EMAIL PROTECTED]
Assunto: RE: [obm-l] Cubo de Rubik

e eu que pensava que era somente eu que me sentia
assimvaleu pela observação..

 --- Artur Costa Steiner [EMAIL PROTECTED]
escreveu:  
 
 -Original Message-
 From: [EMAIL PROTECTED]
 [mailto:[EMAIL PROTECTED] On
 Behalf Of [EMAIL PROTECTED]
 Sent: Saturday, February 07, 2004 2:36 PM
 To: [EMAIL PROTECTED]
 Subject: Re: [obm-l] Cubo de Rubik
 
 Ola, 
 
 Fui eu quem enviou o problema dos bispos, resolvido
 brilhantemente pelo
 Nicolau. Eh uma pena que pouquissimas pessoas nesta
 lista se interessam por
 puzzles mecanicos de matematica. Haja vista que
 enviei um site de um cubo
 Rubick em 4 dimensoes e eu pedia para alguem me
 explicar como seria um de 5
 dimensoes e ninguem quis responder.
 
 Eh sem duvida um assunto muito bonito. O que
 acontece e que a matematica eh
 muito extensa eh a tendencia eh que cada um responda
 ou envie mensagens
 novas quando se trata de assunto de que goste mais
 ou domine mais. Ateh
 mesmo por uma questao de tempo. Acho que a grande
 maioria dos participantes
 desta lista nao podem se dedicar aa matematica da
 forma como gostariam.
 Mesmo os maiores experts em Analise Complexa, Teoria
 dos Numeros, Integral
 de Lebesgue, etc tem que se preocupar em fazer valer
 a prosaica desigualdae
 Orcamento = Despesas, o que pode ser mais dificil
 do que provar a hipotese
 de Riemann.
 
 Eu mesmo jah enviei a esta lista uma 20 mensagens
 que ninguem comentou. Eh
 um tanto frustrante, mas o assunto, certamente, nao
 pareceu interessante aos
 outros participantes. 
 O que eu realmente acho muito desagradavel eh quando
 alguem pede ajuda com
 algum problema, voce tenta ajudar e o principal
 interessado nao mais se
 manifesta. Fica-se com a impressao de se ter perdido
 tempo para nada.
 
 Artur
 
 
 
 


=
 Instruções para entrar na lista, sair da lista e
 usar a lista em
 http://www.mat.puc-rio.br/~nicolau/olimp/obm-l.html


= 

__

Yahoo! GeoCities: 15MB de espaço grátis para criar seu web site!
http://br.geocities.yahoo.com/

=
Instruções para entrar na lista, sair da lista e usar a lista em
http://www.mat.puc-rio.br/~nicolau/olimp/obm-l.html

=


=
Instruções para entrar na lista, sair da lista e usar a lista em
http://www.mat.puc-rio.br/~nicolau/olimp/obm-l.html
=


[obm-l] Cubo de Rubik

2004-02-07 Por tôpico Douglas Ribeiro Silva








Olá pessoal, sei que não mando
e-mails freqüentemente pra lista mas sempre estou lendo os enviados por vocês...



Recentemente num dos e-mails que teve como assunto Extremamente
Difícil que tratava sobre o problema dos bispos alguém citou o WinArc, que por sinal achei um
software bem interessante pra matemática recreativa. Vi que o programa trazia os
cubos de Rubik, de 1x1x1(sem propósito?!) até o 5x5x5, passando pelo famoso 3x3x3(vulgo cubo mágico).
Fiquei curioso pra saber o numero de possíveis combinações do tal cubo e
encontrei isso nesta pagina: http://mathworld.wolfram.com/RubiksCube.html

Eles mostram o possível numero de combinações, mas eu
gostaria que alguém se possível explicasse que fundamento foi usado para aquele
cálculo.



A propósito... eu tenho o clássico
3x3x3 e me interessei em adquirir o 4x4x4 e o 5x5x5. Gostaria também de ter um
que é em formato de dodecaedro. Alguém sabe de algum lugar aqui no Brasil que
venda? Eu moro em Recife e aqui já é difícil de encontrar o 3x3x3, os outros eu
nunca vi pra vender pelo menos por aqui. Encontrei paginas no exterior que
vendem, mas se tudo pudesse ser feito em território nacional seria melhor hehehe



Até mais...  Douglas
Ribeiro








[obm-l] Círculo da Morte

2003-12-11 Por tôpico Douglas Ribeiro Silva








Esse eu achei muito interessante... Eu
poderia encurtar tudo mas vou contar a historia como
me foi proposta...



Durante ter vencido uma longa guerra, um
Rei fez como prisioneiros 99 dos guerreiros de seu inimigo. Ele estava disposto
a matá-los, mas não queria tirar suas vidas sem propósito. Arrumou então uma
desculpa de casar sua filha, oferecendo a mão da moça a qualquer príncipe que
aceitasse um desafio proposto por ele. Um certo dia um príncipe vindo de muito
longe chegou ao reino e pediu a mão da moça. Prontamente, o Rei disse que teria
que passar por um desafio e o príncipe aceitou. Então o Rei lhe explicou qual
era a situação: 



Eu tenho 99 prisioneiros de guerra
no calabouço. Irei dispô-los em forma circular, e darei uma espada a um deles.
Logo após disso você irá adentrar no círculo em qualquer lugar que queira. O
homem a receber a espada irá matar o que estiver a sua esquerda e passará a
espada para o próximo a sua esquerda também. Este, que recebeu a espada, fará o
mesmo. Matará o que está a sua esquerda e passará para o próximo, e assim
sucessivamente até sobrar uma única pessoa no círculo. Se você for o último
terá então a mão da minha filha.



a) Considerando o homem que recebeu a espada
como o nº 1, o da sua esquerda o nº 2, e assim por diante, Em que posição do
círculo o príncipe deverá ficar para permanecer vivo?

b) E se o círculo tivesse k
pessoas? Qual o que permaneceria vivo?



Essa aqui não faz parte da questão mas eu fiquei curioso e resolvi propô-la: Se a espada fosse
entregue aleatoriamente para algum dos 99 prisioneiros só depois do príncipe
entrar no círculo, qual a probabilidade dele ficar vivo no final?



Eu resolvi o
a) e o b) na
época que me foram propostos, mas obtive a fórmula geral por tentativas e
queria uma solução mais higiênica. A
outra pergunta que eu propus não soube como resolver.



Abraços, Douglas








[obm-l] RES: [obm-l] Círculo da Morte

2003-12-11 Por tôpico Douglas Ribeiro Silva
É verdade... só que eu sem querer propus errado. Desculpe ehehehhe

Alem do que creio que você se enganou, no caso seria 1/100 porque o
príncipe é o 100º participante do circulo.

Na hora que eu escrevi estava com um pouco de pressa e acabei me
enganando... Corrigindo a proposição da probabilidade:

Se a espada fosse entregue aleatoriamente para algum dos k
prisioneiros só depois do príncipe entrar no círculo, qual a
probabilidade dele ficar vivo no final?





-Mensagem original-
De: [EMAIL PROTECTED] [mailto:[EMAIL PROTECTED] Em
nome de Qwert Smith
Enviada em: quinta-feira, 11 de dezembro de 2003 17:33
Para: [EMAIL PROTECTED]
Assunto: RE: [obm-l] Círculo da Morte

hmmm... a c) pareece facil de responder...tao facil ki deve estar
errado...
vamos supor ki o principe entra na posicao x... essa posicao so
sobrevive se 
o prisioneiro que receber a espada estive em uma outra posicao y
(relativa a 
x)... portanto as chance sao 1/99 de sobreviver, ja que tem 99
prisioneiros 
e so uma resultaria em sucesso para o principe


From: Douglas Ribeiro Silva [EMAIL PROTECTED]
Reply-To: [EMAIL PROTECTED]
To: [EMAIL PROTECTED]
Subject: [obm-l] Círculo da Morte
Date: Thu, 11 Dec 2003 16:53:09 -0300

Esse eu achei muito interessante... Eu poderia encurtar tudo mas vou
contar a historia como me foi proposta...

Durante ter vencido uma longa guerra, um Rei fez como prisioneiros 99
dos guerreiros de seu inimigo. Ele estava disposto a matá-los, mas não
queria tirar suas vidas sem propósito. Arrumou então uma desculpa de
casar sua filha, oferecendo a mão da moça a qualquer príncipe que
aceitasse um desafio proposto por ele. Um certo dia um príncipe vindo
de
muito longe chegou ao reino e pediu a mão da moça. Prontamente, o Rei
disse que teria que passar por um desafio e o príncipe aceitou. Então o
Rei lhe explicou qual era a situação:

“Eu tenho 99 prisioneiros de guerra no calabouço. Irei dispô-los em
forma circular, e darei uma espada a um deles. Logo após disso você irá
adentrar no círculo em qualquer lugar que queira. O homem a receber a
espada irá matar o que estiver a sua esquerda e passará a espada para o
próximo a sua esquerda também. Este, que recebeu a espada, fará o
mesmo.
Matará o que está a sua esquerda e passará para o próximo, e assim
sucessivamente até sobrar uma única pessoa no círculo. Se você for o
último terá então a mão da minha filha.”

a)  Considerando o homem que recebeu a espada como o nº 1, o da sua
esquerda o nº 2, e assim por diante, Em que posição do círculo o
príncipe deverá ficar para permanecer vivo?
b)  E se o círculo tivesse “k” pessoas? Qual o que permaneceria
vivo?

Essa aqui não faz parte da questão mas eu fiquei curioso e resolvi
propô-la: Se a espada fosse entregue aleatoriamente para algum dos 99
prisioneiros só depois do príncipe entrar no círculo, qual a
probabilidade dele ficar vivo no final?

Eu resolvi o a) e o b) na época que me foram propostos, mas obtive a
fórmula geral por tentativas e queria uma solução mais “higiênica”. A
outra pergunta que eu propus não soube como resolver.

Abraços, Douglas

_
Take advantage of our best MSN Dial-up offer of the year — six months 
@$9.95/month. Sign up now! http://join.msn.com/?page=dept/dialup


=
Instruções para entrar na lista, sair da lista e usar a lista em
http://www.mat.puc-rio.br/~nicolau/olimp/obm-l.html

=


=
Instruções para entrar na lista, sair da lista e usar a lista em
http://www.mat.puc-rio.br/~nicolau/olimp/obm-l.html
=


RES: [obm-l] geometria2

2003-11-17 Por tôpico Douglas Ribeiro Silva
Esse problema caiu na UFPE em 98 ou 99 creio eu... Faltou um dado:
m(BÂC) = 20°

-Mensagem original-
De: [EMAIL PROTECTED]
[mailto:[EMAIL PROTECTED] Em nome de Rogerio Ponce
Enviada em: segunda-feira, 17 de novembro de 2003 20:24
Para: [EMAIL PROTECTED]
Assunto: Re: [obm-l] geometria2

Olá Bruno,
não dá mesmo p/ resolver , pois faltam dados.
Conforme as condições dadas , ADE pode ser construído 0 .
Mas , sem fugir ao enunciado , pode-se fazer ABC=60 , o que tornaria
ADE=0 .
Portanto ADE não é fixo .
[]´s
Rogério


Olá a todos,
Estou com mais um probleminha de geometria, que não consigo resolver
Desta vez acho q dá um número bonitinho.
Aceito qualquer sugestão...
Até,
Bruno
 geometria2.gif 

_
MSN Messenger: converse com os seus amigos online.  
http://messenger.msn.com.br


=
Instruções para entrar na lista, sair da lista e usar a lista em
http://www.mat.puc-rio.br/~nicolau/olimp/obm-l.html

=

=
Instruções para entrar na lista, sair da lista e usar a lista em
http://www.mat.puc-rio.br/~nicolau/olimp/obm-l.html
=


[obm-l] Parabola estreita?!

2003-11-13 Por tôpico Douglas Ribeiro Silva








Alguem saberia me
dizer por favor o que é uma trajetória parabólica
estreita? Vi esse termo num texto e não consegui compreender sobre que tipo de parábola
se tratava. Também não sei se existe realmente a tal parábola estreita pois era um texto humorístico.








[obm-l] RES: [obm-l] RES: [obm-l] RES: [obm-l] Área da Lua

2003-10-26 Por tôpico Douglas Ribeiro Silva
Desculpe então Marcos, pois só faço parte da lista desde a segunda
semana de setembro e como eu havia dito não sabia se o problema já tinha
aparecido ou não.

Não estou decepcionado pela aparição das funções inversas, só queria ter
certeza que seria necessário usá-las na resposta final do problema.
Quanto a notação que usei para elas, realmente admito que não é a
correta, mas creio que deu pra entender o sentido que eu quis passar.

A propósito, obrigado por ratificar o aparecimento das funções
trigonométricas inversas.

Um abraço, Douglas Ribeiro Silva

-Mensagem original-
De: [EMAIL PROTECTED]
[mailto:[EMAIL PROTECTED] Em nome de Marcos
Enviada em: sábado, 25 de outubro de 2003 22:31
Para: [EMAIL PROTECTED]
Assunto: [obm-l] RES: [obm-l] RES: [obm-l] Área da Lua

Antes de começar eu não sou o Marcos que é citado na resposta ..
Também não vou resolver porque pelo jeito o problema já foi resolvido na
lista pelo Cláudio.
Eu soh queria mesmo comentar que esta eh a quinta vez que esse problema
me aparece ESSE ano (nunca tinha aparecido antes...) e nas quatro vezes
anteriores quando eu mostrei a solução TODOS fizeram a mesma pergunta
como se tivessem ficado decepcionados pela aparição das funções
trigonométricas inversas.
Somente um detalhe POQ e PBQ eh q são os arcos cujos
cossenos/senos/tangentes são descobertos no meio das contas feitas pra
resolver o problema .. não tem muito sentido Arctan(PBQ) como está
escrito na sua msg.. o q faz sentido eh Arctan(x) = PBQ.

[]'s MP

-Mensagem original-
De: [EMAIL PROTECTED]
[mailto:[EMAIL PROTECTED] Em nome de Douglas Ribeiro
Silva
Enviada em: sábado, 25 de outubro de 2003 22:10
Para: [EMAIL PROTECTED]
Assunto: [obm-l] RES: [obm-l] Área da Lua

Marcos, fiz uma breve figura no PaintBrush mesmo mas espero que dê para
entender o propósito da questão...

http://www.klystron.kit.net/lua.jpg

A área da Lua que eu citei é a área que está em cinza.

A propósito Cláudio... a resposta final do problema tem realmente que
ficar em função do arccos/sen/tg de POQ e PBQ?

Abraços, Douglas Ribeiro

-Mensagem original-
De: [EMAIL PROTECTED]
[mailto:[EMAIL PROTECTED] Em nome de Marcos Braga
Enviada em: sábado, 25 de outubro de 2003 18:32
Para: [EMAIL PROTECTED]
Assunto: Re: [obm-l] Área da Lua

Douglas ,

Eu não estou conseguindo visualizar a fugura , digo, um arco de 
circunferência de A para C com
centro em B , teria como me ajudar ou enviar a figura ou um site que
tnha a 
mesma ?

Abçs ,

Marcos



At 03:01 25/10/2003 -0300, you wrote:
Esse problema me foi passado há algum tempo mas não consegui uma
solução
sucinta para ele. Não sei se o problema já foi discutido na lista, mas
lá vai...

Seja um quadrado ABCD de lado a. Inscreve-se no quadrado uma
circunferencia. Traça-se um arco de circunferência de A para C com
centro em B. Este arco intercepta a circunferência inscrita em 2
pontos.
Qual a área dessa figura em forma de Lua?

Não me lembro bem mas acho que alguém me disse certa vez que esse
problema poderia ser feito de 2 maneiras, uma por geometria plana,
outra
por integral. Se possível gostaria de saber os 2 métodos.

Abraços, Douglas.

===
==
Instruções para entrar na lista, sair da lista e usar a lista em
http://www.mat.puc-rio.br/~nicolau/olimp/obm-l.html
===
==



=
Instruções para entrar na lista, sair da lista e usar a lista em
http://www.mat.puc-rio.br/~nicolau/olimp/obm-l.html

=


=
Instruções para entrar na lista, sair da lista e usar a lista em
http://www.mat.puc-rio.br/~nicolau/olimp/obm-l.html

=

---
Incoming mail is certified Virus Free.
Checked by AVG anti-virus system (http://www.grisoft.com).
Version: 6.0.512 / Virus Database: 309 - Release Date: 19/8/2003
 

---
Outgoing mail is certified Virus Free.
Checked by AVG anti-virus system (http://www.grisoft.com).
Version: 6.0.512 / Virus Database: 309 - Release Date: 19/8/2003
 


=
Instruções para entrar na lista, sair da lista e usar a lista em
http://www.mat.puc-rio.br/~nicolau/olimp/obm-l.html

=

=
Instruções para entrar na lista, sair da lista e usar a lista em
http://www.mat.puc-rio.br/~nicolau/olimp/obm-l.html
=


[obm-l] Área da Lua

2003-10-25 Por tôpico Douglas Ribeiro Silva
Esse problema me foi passado há algum tempo mas não consegui uma solução
sucinta para ele. Não sei se o problema já foi discutido na lista, mas
lá vai...

Seja um quadrado ABCD de lado a. Inscreve-se no quadrado uma
circunferencia. Traça-se um arco de circunferência de A para C com
centro em B. Este arco intercepta a circunferência inscrita em 2 pontos.
Qual a área dessa figura em forma de Lua?

Não me lembro bem mas acho que alguém me disse certa vez que esse
problema poderia ser feito de 2 maneiras, uma por geometria plana, outra
por integral. Se possível gostaria de saber os 2 métodos.

Abraços, Douglas.

=
Instruções para entrar na lista, sair da lista e usar a lista em
http://www.mat.puc-rio.br/~nicolau/olimp/obm-l.html
=


[obm-l] RES: [obm-l] Área da Lua

2003-10-25 Por tôpico Douglas Ribeiro Silva
Marcos, fiz uma breve figura no PaintBrush mesmo mas espero que dê para
entender o propósito da questão...

http://www.klystron.kit.net/lua.jpg

A área da Lua que eu citei é a área que está em cinza.

A propósito Cláudio... a resposta final do problema tem realmente que
ficar em função do arccos/sen/tg de POQ e PBQ?

Abraços, Douglas Ribeiro

-Mensagem original-
De: [EMAIL PROTECTED]
[mailto:[EMAIL PROTECTED] Em nome de Marcos Braga
Enviada em: sábado, 25 de outubro de 2003 18:32
Para: [EMAIL PROTECTED]
Assunto: Re: [obm-l] Área da Lua

Douglas ,

Eu não estou conseguindo visualizar a fugura , digo, um arco de 
circunferência de A para C com
centro em B , teria como me ajudar ou enviar a figura ou um site que
tnha a 
mesma ?

Abçs ,

Marcos



At 03:01 25/10/2003 -0300, you wrote:
Esse problema me foi passado há algum tempo mas não consegui uma
solução
sucinta para ele. Não sei se o problema já foi discutido na lista, mas
lá vai...

Seja um quadrado ABCD de lado a. Inscreve-se no quadrado uma
circunferencia. Traça-se um arco de circunferência de A para C com
centro em B. Este arco intercepta a circunferência inscrita em 2
pontos.
Qual a área dessa figura em forma de Lua?

Não me lembro bem mas acho que alguém me disse certa vez que esse
problema poderia ser feito de 2 maneiras, uma por geometria plana,
outra
por integral. Se possível gostaria de saber os 2 métodos.

Abraços, Douglas.

===
==
Instruções para entrar na lista, sair da lista e usar a lista em
http://www.mat.puc-rio.br/~nicolau/olimp/obm-l.html
===
==



=
Instruções para entrar na lista, sair da lista e usar a lista em
http://www.mat.puc-rio.br/~nicolau/olimp/obm-l.html

=

=
Instruções para entrar na lista, sair da lista e usar a lista em
http://www.mat.puc-rio.br/~nicolau/olimp/obm-l.html
=


RES: RES: [obm-l] Geometria (Mr. Crowley)

2003-10-24 Por tôpico Douglas Ribeiro Silva
Cesar, não entendi se você queria saber a prova do fato de serem
retângulos, ou de serem semelhantes, em todo caso estou enviando tudo...

Prova-se que CFB é retângulo pelo fato de todo triangulo retângulo estar
inscrito numa semi-circunferencia, onde o diâmetro da
semi-circunferencia é a hipotenusa do triangulo retângulo, nesse caso BC
é a hipotenusa, CF e FB os catetos. F é ângulo reto já que BÔC(Considere
O ponto médio de BC e centro da circunferencia) vale 180°, e como F está
sobre a circunferencia então CFB é metade de BÔC.

Prova-se que DCB é retângulo simplesmente pelo enunciado da questão, já
que ele diz que os triângulos são retângulo-isosceles. Logo, ACB = 45° e
BCA = 45º, então DCB = 90°

Para provar a semelhança dos 2 triangulos usa-se o fato deles terem em
comum o ângulo de 90° e o ângulo CBF, já que F está contido no segmente
BD, então CBF = CBD = arctg(1/2)

Se tiver faltando alguma coisa, ou estiver algo errado, avise-me por
favor.

[]'s Douglas


-Mensagem original-
De: [EMAIL PROTECTED]
[mailto:[EMAIL PROTECTED] Em nome de Cesar Ryudi
Kawakami
Enviada em: sexta-feira, 24 de outubro de 2003 13:46
Para: [EMAIL PROTECTED]
Assunto: Re: RES: [obm-l] Geometria (Mr. Crowley)

At 02:01 24/10/2003, you wrote:
Se a circunferência tem diâmetro BC então o centro dela está no ponto
médio de BC. (Creio que foi uma mera desatenção sua Cesar)

Eu pensei nessa hipótese, e foi mera desatenção de minha parte mesmo...

CÁLCULO DE DF:

Como F é a intersecção da circunferência com BD, então o triangulo CFB
é
retângulo. Nota-se que o triangulo DCB também é retângulo.

Como você provou isso? Eu desenhei e também tive essa conclusão, mas não

pude provar isso de modo satisfatório...

Um abraço,

Cesar Ryudi Kawakami 


=
Instruções para entrar na lista, sair da lista e usar a lista em
http://www.mat.puc-rio.br/~nicolau/olimp/obm-l.html

=

=
Instruções para entrar na lista, sair da lista e usar a lista em
http://www.mat.puc-rio.br/~nicolau/olimp/obm-l.html
=


RES: [obm-l] Geometria (Mr. Crowley)

2003-10-23 Por tôpico Douglas Ribeiro Silva
Bom, espero que eu não tenha errado, mas se encontrarem alguma falha,
favor avisem...

Item C:

Se a circunferência tem diâmetro BC então o centro dela está no ponto
médio de BC. (Creio que foi uma mera desatenção sua Cesar)

CÁLCULO DE DF:

Como F é a intersecção da circunferência com BD, então o triangulo CFB é
retângulo. Nota-se que o triangulo DCB também é retângulo. Como os 2
triângulos citados são semelhantes(Possuem em comum o ângulo reto, e o
ângulo B, logo o outro também é igual) pode-se aplicar uma regra de 3
simples: BD/BC = BC/BF (Uma das relações notáveis do triangulo retângulo
geralmente mostrada como c² = a.m).

BC² = BD.BF

2a² = (sqrt(10).a/2).BF

BF = sqrt(10)2a/5

RESPOSTA:

DF = (BD - BF)
   = sqrt(10).a/2 - sqrt(10)2a/5

LOGO DF = sqrt(10).a/10


CÁLCULO DE EF:

Como CD/AD = 2, e percebe-se que os triangulos ADE e CEB são
semelhantes, então BE/ED = 2, Logo BE é BD/3

RESPOSTA:

EF = BF - BE
   = sqrt(10)2a/5 - sqrt(10).a/6

LOGO: EF = sqrt(10)7a/30




-Mensagem original-
De: [EMAIL PROTECTED]
[mailto:[EMAIL PROTECTED] Em nome de Cesar Ryudi
Kawakami
Enviada em: quinta-feira, 23 de outubro de 2003 22:19
Para: [EMAIL PROTECTED]
Assunto: Re: [obm-l] Geometria (Mr. Crowley)

Prolongando BA e sendo M o pé da distância de D em relação à AB, e sendo
N 
o pé da altura de D em relação à AC, teremos um quadrado de lado a/2
AMDN, 
pois CAD = 45º e ADN = 90º/2 = 45º, sendo ADN um triângulo isósceles de 
catetos a/2 (NDC congruente a ADN)

Resolução item A: Assim, BM = 3a/2, e DM = a/2. Aplicando o Teorema de 
Pitágoras sobre o triângulo DMB temos que DB = sqrt(10).a/2.

Retomando o fato de AMDN ser um quadrado, BM // DN. Como NDE = EBA 
(alternos internos), e AEB = DEN (opostos pelo vértice), os triângulos
ABE 
e EDN são semelhantes. Colocando em proporção os lados homólogos, temos:

AB/DN = AE/EN, ou, então,

2 = AE/EN

Logo, 2(EN) = AE, e AE = 2(AN)/3. Assim, AE = AC/3 = a/3.

Resolução item B: Aplicando pitágoras sobre o triângulo BAE, temos que
BE = 
sqrt(10).a/3.
Subtraindo, temos que DE = sqrt(10).a/6

O enunciado do C eu não entendi...

circunferência de diâmetro BC, mas centro onde?

Um abraço,

Cesar Ryudi Kawakami

At 03:05 23/10/2003, you wrote:
Olá Pessoal,

Me ajudem nesta questaum:

Sejam ABC e ACD dois triângulos retângulos isósceles
com o lado AC comum, e os vértices B e D situados em
semiplanos distintos em relação ao lado AC. Nestes
triângulos AB = AC = a e AD = CD.

a) Calcule a diagonal BD, do quadrilátero ABCD.
b) Seja E o ponto de interseção de AC com BD. Calcule
BE e ED.
c) Seja F a interseção da circunferência de diâmetro BC
com a diagonal BD. Calcule DF e EF.


Grato

Mr. Crowley

___
___
Acabe com aquelas janelinhas que pulam na sua tela.
AntiPop-up UOL - É grátis!
http://antipopup.uol.com.br/


===
==
Instruções para entrar na lista, sair da lista e usar a lista em
http://www.mat.puc-rio.br/~nicolau/olimp/obm-l.html
===
==


=
Instruções para entrar na lista, sair da lista e usar a lista em
http://www.mat.puc-rio.br/~nicolau/olimp/obm-l.html

=

=
Instruções para entrar na lista, sair da lista e usar a lista em
http://www.mat.puc-rio.br/~nicolau/olimp/obm-l.html
=


[obm-l] Provas da OBM

2003-10-22 Por tôpico Douglas Ribeiro Silva








Olá pessoal, gostaria de saber se
algum de vocês sabe me dizer onde há para download na
Internet as provas da OBM anteriores ao ano de 1998. Eu realmente ficaria muito
grato. Abraços...